NREMT

Hypoglycemia and acute ischemic stroke can present similarly because:
� A:both oxygen and glucose are needed for brain function.
� B:the majority of stroke patients have a history of diabetes.
� C:the most common cause of a stroke is hypoglycemia.
� D:the

� A:both oxygen and glucose are needed for brain function.
Reason: Although stroke and hypoglycemia are two distinctly different conditions, their signs and symptoms are often similar. This is because the brain requires both oxygen and glucose to function

When dealing with an emotionally disturbed patient, you should be MOST concerned with:
� A:gathering all of the patient's medications.
� B:safely transporting to the hospital.
� C:whether the patient could harm you.
� D:obtaining a complete medical histor

You selected C; This is correct!
Reason: When managing any patient with an emotional or psychiatric crisis, your primary concern is your own safety. Safely transporting the patient to the hospital is your ultimate goal. If possible, you should attempt to

You are at the scene where a man panicked while swimming in a small lake. Your initial attempt to rescue him should include:
� A:rowing a small raft to the victim.
� B:reaching for the victim with a long stick.
� C:throwing a rope to the victim.
� D:swimm

You selected B; This is correct!
Reason: General rules to follow when attempting to rescue a patient from the water include "reach, throw, row, and then go." In this case, you should attempt to reach the victim by having him grab hold of a large stick or

How should you classify a patient's nature of illness if he or she has a low blood glucose level, bizarre behavior, and shallow breathing?
� A:Behavioral emergency
� B:Altered mental status
� C:Respiratory emergency
� D:Cardiac compromise

The correct answer is B;
Reason: The nature of illness (NOI) is the medical equivalent to mechanism of injury (MOI). Altered mental status should be the suspected NOI in any patient with any fluctuation in level of consciousness, which can range from biza

A young female is unresponsive after overdosing on an unknown type of drug. Her respirations are slow and shallow and her pulse is slow and weak. Which of the following drugs is the LEAST likely cause of her condition?
� A:Seconal
� B:Heroin
� C:Cocaine

The correct answer is C;
Reason: Of the drugs listed, cocaine would be the least likely cause of the patient's condition. Cocaine is a central nervous system (CNS) stimulant; you would expect her to be hypertensive, tachycardic, tachypneic, and perhaps ev

Activated charcoal is contraindicated for a patient who is:
� A:conscious and alert and has ingested a large amount of Motrin.
� B:emotionally upset and has ingested two bottles of aspirin.
� C:agitated and claims to have ingested a bottle of Tylenol.
� D

You selected D; This is correct!
Reason:Activated charcoal adsorbs (sticks to) many ingested substances, preventing them from being absorbed into the body by the stomach or intestines. In some cases, you may give activated charcoal to patients who have in

The MOST obvious way to reduce heat loss from radiation and convection is to:
� A:move away from a cold object.
� B:increase metabolism by shivering.
� C:wear a thick wind-proof jacket.
� D:move to a warmer environment.

you selected D; This is correct!
Reason:In a cold environment, the body has two ways of staying warm: generating heat (thermogenesis) and reducing heat loss. Radiation is the transfer of heat by radiant energy. The body can lose heat by radiation, such as

A near-drowning is MOST accurately defined as:
� A:complications within 24 hours following submersion in water.
� B:immediate death due to prolonged submersion in water.
� C:survival for at least 24 hours following submersion in water.
� D:death greater t

You selected C; This is correct!
Reason:Collectively, drowning and near-drowning are referred to as submersion injuries. Drowning is defined as death after submersion in a liquid medium, usually water. In a drowning, death is either immediate or occurs wi

A young woman reports significant weight loss over the last month, persistent fever, and night sweats. When you assess her, you note the presence of dark purple lesions covering her trunk and upper extremities. You should suspect:
� A:end-stage cancer.

You selected B; This is correct!
Reason:Weight loss, fever, and night sweats could indicate tuberculosis or HIV/AIDS; however, the dark purple lesions on the skin, which are called Kaposi's sarcoma, are malignant skin tumors and are a classic finding in p

A 48-year-old male became acutely hypoxic, experienced a seizure, and is now postictal. The MOST effective way to prevent another seizure is to:
� A:dim the lights in the back of the ambulance.
� B:place him in the recovery position.
� C:administer high-f

You selected C; This is correct!
Reason:You should administer high-flow oxygen to all patients who are actively seizing and to patients who experienced a seizure and are postictal. This is especially true if the seizure was caused by hypoxia. Increasing t

Which of the following conditions would be the LEAST likely to be present in a patient who was submerged in water?
� A:Spinal injury
� B:Gastric distention
� C:Hyperglycemia
� D:Laryngospasm

You selected C; This is correct!
Reason: Many factors can contribute to or result from a submersion injury (eg, drowning, near-drowning). It is not uncommon for a person to experience a spinal injury after diving head first into shallow water, especially

A 42-year-old male presents with fever, a severe headache, and a stiff neck. He is conscious, but confused. His wife tells you that he does not have any medical problems and does not take any medications. You should be MOST suspicious for:
� A:acute strok

You selected C; This is correct!
Reason:Meningitis is an inflammation of the protective coverings of the brain and spinal cord (meninges). Common signs and symptoms of meningitis include fever, headache, neck stiffness (nuchal rigidity), and vomiting. An

All of the following are signs of gastrointestinal bleeding, EXCEPT:
� A:melena.
� B:hematemesis.
� C:tachycardia.
� D:hemoptysis.

You selected D; This is correct!
Reason:Signs and symptoms of gastrointestinal (GI) bleeding include abdominal pain; vomiting blood (hematemesis); the passage of dark, tarry stools (melena); and bright red rectal bleeding (hematochezia). If blood loss is

A 24-year-old female presents with a rash to her left leg and swollen, painful knee joints. She tells you that she and her friends returned from a hiking trip in the mountains a week ago. She is conscious and alert with a blood pressure of 112/62 mm Hg, a

You selected C; This is correct!
Reason:The patient's symptoms and her history of a recent hiking trip are consistent with Lyme disease, which was the result of a tick bite. Ticks can carry two infectious diseases: Lyme disease and Rocky Mountain spotted

A 50-year-old woman who is conscious and alert complains of a severe migraine headache. When caring for her, you should generally avoid:
� A:transporting her in a supine position.
� B:shining a light into her pupils.
� C:dimming the lights in the ambulanc

You selected B; This is correct!
Reason:Patients with migraine or cluster headaches typically have photophobia (light sensitivity). Any type of bright light, especially if shone directly into the eyes, will cause the patient with a headache unnecessary se

A patient who overdosed on heroin would be expected to present with:
� A:tachycardia.
� B:hyperpnea.
� C:hypotension.
� D:dilated pupils.

The correct answer is C;
Reason:Heroin is a Schedule I (illegal) narcotic that is typically injected. As with all narcotics, legal or illegal, overdose causes depression of the central nervous system (CNS), resulting in a decreased level of consciousness;

When caring for any patient with a decreased level of consciousness, your primary concern should be the:
� A:potential for airway compromise.
� B:patient's blood glucose level.
� C:possibility of a spinal injury.
� D:possibility of a drug overdose.

You selected A; This is correct!
Reason:Altered mental status could be caused by a high or low blood glucose level, drug overdose, or head injury, among other causes. Furthermore, the possibility of a spinal injury should be considered if the patient was

A 16-year-old, 125-pound male ingested a bottle of aspirin approximately 20 minutes ago. Medical control orders you to administer activated charcoal in a dose of 1 g/kg. How much activated charcoal should you administer?
� A:54 g
� B:60 g
� C:51 g
� D:57

You selected D; This is correct!
Reason: First, you must determine the patient's weight in kilograms (kg). Either of the following formulae can be used to convert pounds to kilograms: Formula 1: weight (in pounds) � 2.2 = weight in kg. Formula 2: weight (

Which of the following is the MOST significant finding in a patient with a severe headache?
� A:Pain in both legs
� B:Chest discomfort
� C:Unilateral weakness
� D:Abdominal tenderness

You selected C; This is correct!
Reason: Unilateral weakness (weakness on one side of the body) is a clinically significant finding in a patient with a headache because it could indicate a stroke (ischemic or hemorrhagic). Abdominal, chest, and leg pain a

When caring for a patient with severe hypothermia who is in cardiac arrest, you should:
� A:perform rescue breathing only.
� B:perform BLS and transport.
� C:avoid using the AED.
� D:hyperventilate the patient.

The correct answer is B;
Reason:Patients with severe hypothermia (core body temperature < 86�F) who are in cardiac arrest should be managed with basic life support (chest compressions and ventilations), passive external rewarming (ie, removal of wet cloth

You should not attempt to actively rewarm a patient with moderate or severe hypothermia in the field because:
� A:rewarming too quickly can cause a fatal cardiac dysrhythmia.
� B:the risk of inadvertently inducing hyperthermia is too high.
� C:it is painf

You selected A; This is correct!
Reason:When caring for a patient with hypothermia, your goal is to prevent further heat loss; this involves removing wet clothing, applying warm blankets, and allowing the patient's body temperature to rise gradually and n

You respond to a grocery store where a 39-year-old man reportedly experienced a seizure. When you arrive at the scene, a clerk begins to escort you to the patient. She tells you that the man stopped seizing about 5 minutes ago. If the patient truly experi

You selected D; This is correct!
Reason: Most seizures are followed by a postictal phase, in which the patient is unresponsive at first and then gradually regains consciousness. In most cases, the postictal patient appears dazed, confused, or disoriented;

You are called to a local park for an ill person. It is a hot day and the humidity is high. When you arrive, a bystander directs you to the patient, a young male who is semiconscious. His skin is flushed, hot, and moist. Your FIRST action in the managemen

You selected D; This is correct!
Reason:Your first action in a heat-related emergency is to move the patient to a cooler environment. Once you have moved the patient to a cooler place, you should begin your assessment and treat the patient accordingly. Re

A known diabetic female is found unresponsive. Her respirations are rapid and shallow; her skin is cool, clammy, and pale; and her pulse is rapid and weak. Which of the following would BEST explain the likely cause of her condition?
� A:Failure to take in

The correct answer is B;
Reason:The patient has classic signs of insulin shock, a condition caused by a low blood glucose level (hypoglycemia). Common causes of insulin shock include insulin overdose (accidental or intentional), failure to eat (or not eat

You are dispatched to a residence for a 20-year-old male with respiratory distress. When you arrive, you find that the patient has a tracheostomy tube and is ventilator dependent. His mother tells you that he was doing fine, but then suddenly began experi

you selected C; This is correct!
Reason:If a ventilator-dependent patient experiences a sudden onset of respiratory distress, you should first remove him or her from the mechanical ventilator and begin manual ventilation with a bag-mask device; if the pat

A middle-aged man was found floating facedown in a small pond. When you arrive at the scene, bystanders are present, but nobody has removed him from the water because they thought he was dead. After reaching the victim, you should:
� A:float a buoyant bac

The correct answer is B;
Reason:After safely reaching a drowning victim, you should first turn him to a supine position by rotating the entire upper half of the body as a single unit; protect the cervical spine as you do this because a spinal injury canno

Which of the following patients with diabetes is the BEST candidate for oral glucose?
� A:A confused patient who has cool, clammy skin
� B:A semiconscious patient with pale, clammy skin
� C:An unresponsive patient who took too much insulin
� D:A confused

You selected A; This is correct!
Reason:Oral glucose is indicated for patients with diabetes who are in insulin shock (hypoglycemic crisis), or for any patient with symptomatic hypoglycemia (as documented by glucometer). The patient must be conscious and

A 40-year-old woman presents with widespread hives that she noticed about 45 minutes after taking penicillin. She is conscious and alert and denies difficulty breathing. Her breath sounds are clear to auscultation bilaterally, her vital signs are stable,

You selected C; This is correct!
Reason:The patient is experiencing an allergic reaction, but she is not in anaphylactic shock. Urticaria (hives) is common to all allergic reactions, regardless of severity; however, wheezing and hypotension, which she doe

Immediately following a generalized motor seizure, most patients are:
� A:hyperactive.
� B:confused.
� C:apneic.
� D:awake and alert.

You selected B; This is correct!
Reason:After a generalized (grand mal) motor seizure, the patient typically will be confused, sleepy, or in some cases, combative. This is referred to as the postictal phase. The patient's level of consciousness typically

When assessing a conscious patient who overdosed on a drug, you should FIRST determine:
� A:if there is a history of prior overdose.
� B:the patient's weight in kilograms.
� C:the type of medication ingested.
� D:when the medication was ingested.

You selected C; This is correct!
Reason: When assessing a patient who has overdosed on a drug, you should first determine what was ingested, which will provide you with immediate information about whether or not the substance is toxic. You should then fin

Which of the following actions should be carried out during the primary assessment of an unresponsive patient?
� A:Assessing the skin
� B:Obtaining a blood pressure
� C:Palpating the cranium
� D:Auscultating the lungs

You selected A; This is correct!
Reason:Components of the primary assessment for both responsive and unresponsive patients include assessing and managing the airway and assessing and managing circulation, which includes controlling any major bleeding; ass

When restraining a violent patient, you should make sure that:
� A:the patient is restrained using maximal force.
� B:at least two EMTs restrain the patient.
� C:consent for restraint has been obtained from a family member.
� D:someone talks to the patien

You selected D; This is correct!
Reason:If a violent patient needs to be restrained, you must ensure the presence of at least four people (one per extremity). One of the EMTs should continuously talk to the patient to explain what is happening, even if th

Immediately following a generalized seizure involving tonic-clonic activity, most patients:
� A:have a slow heart rate because of cardiac depression during the seizure.
� B:are unresponsive but gradually regain consciousness.
� C:experience prolonged apne

You selected B; This is correct!
Reason:Most generalized (grand mal) seizures last 3 to 5 minutes and are followed by a lengthy (5 to 30 minutes) period called the postictal phase, in which the patient is unresponsive at first but gradually regains consci

Which of the following organs are contained within the right upper quadrant of the abdomen?
� A:Liver and gallbladder
� B:Stomach and gallbladder
� C:Liver and spleen
� D:Liver and stomach

You selected A; This is correct!
Reason:The liver and gallbladder lie within the right upper quadrant of the abdomen. Most of the stomach is within the left upper quadrant, as is the entire spleen.

You respond to a baseball field for a person who was struck by lightning. When you arrive, you see one patient who appears confused and is ambulatory; a second patient who is conscious, sitting on the ground, and holding his arm; and a third patient who i

You selected D; The correct answer is A;
Reason:The process of triaging multiple victims of a lightning strike is different than the conventional triage methods used during a mass-casualty incident. When a person is struck by lightning, respiratory or car

A 32-year-old man who was stung by a bee has diffuse hives, facial swelling, and difficulty breathing. When he breathes, you hear audible stridor. What does this indicate?
� A:Narrowing of the two mainstem bronchi
� B:Swelling of the upper airway structur

You selected B; This is correct!
Reason:This patient is experiencing a severe allergic reaction (anaphylaxis). Stridor, which is a high-pitched sound heard on inhalation, indicates swelling of the structures and tissues of the upper airway. If not promptl

A behavioral crisis is MOST accurately defined as:
� A:any reaction to an event that interferes with the activities of daily living or has become unacceptable to the patient, family, or community.
� B:an acute psychiatric emergency characterized by violen

You selected A; This is correct!
Reason:The definition of a behavioral crisis or emergency is any reaction to an event that interferes with the activities of daily living (eg, bathing, dressing, eating) or has become unacceptable to the patient, his or he

You are called to an assisted living center where an attendant found a 72-year-old man unresponsive. The patient had recent hip surgery and has been taking Vicodin for pain. His respirations are slow and shallow and his pulse is slow and weak. You should:

You selected C; This is correct!
Reason:Vicodin is a combination of hydrocodone and acetaminophen (APAP), the active ingredient in Tylenol. Hydrocodone is a potent narcotic analgesic. When taken in excess, it can suppress the central nervous system and ca

In the patient with diabetes, insulin shock typically presents with:
� A:clammy skin and a rapid onset.
� B:dry skin and a slow onset.
� C:dry skin and a rapid onset.
� D:clammy skin and a slow onset.

The correct answer is A;
Reason:In the patient with diabetes, insulin shock (hypoglycemic crisis) presents with cool, clammy skin and a rapid onset. The brain is critically dependent on glucose and responds quickly when the body is in short supply. Diabet

Your FIRST action in managing a patient with an altered mental status should be to:
� A:determine if the patient is breathing adequately.
� B:administer one tube of oral glucose.
� C:administer high-flow supplemental oxygen.
� D:ask a family member how th

You selected A; This is correct!
Reason:When treating a patient with an altered mental status, you must first ensure a patent airway and determine if the patient is breathing adequately. If the patient is breathing adequately, administer supplemental oxyg

A patient who overdosed on methamphetamine would be expected to have all of the following clinical signs, EXCEPT:
� A:agitation.
� B:hypertension.
� C:dilated pupils.
� D:bradycardia.

You selected C; The correct answer is D;
Reason:Methamphetamine, an upper, stimulates the central nervous system, causing it to release excessive amounts of adrenaline (epinephrine). Epinephrine increases heart rate and blood pressure; therefore, the pati

A 50-year-old woman with a history of epilepsy is actively seizing. Care for this patient should focus primarily on:
� A:protecting her from injury and ensuring adequate ventilation.
� B:administering high-flow oxygen and requesting an ALS ambulance.
� C:

You selected A; This is correct!
Reason:Seizure deaths are most frequently the result of hypoxia. When a person is actively seizing, he or she is not breathing adequately. Your primary focus when treating a seizure patient is to protect him or her from in

When you arrive at a residence for a man who is "not acting right," you enter the house and find him sitting on his couch. Which of the following findings would be MOST indicative of an altered mental status?
� A:Slurred speech.
� B:Tired appearance.
� C:

You selected A; This is correct!
Reason:Often, an altered mental status can be difficult to assess, especially if you do not know how the patient normally acts. However, there are key findings that should increase your index of suspicion. An abnormal spee

Hypoxia-induced unresponsiveness during a submersion injury is usually the result of:
� A:a cardiac dysrhythmia.
� B:associated hypothermia.
� C:laryngospasm.
� D:water in the lungs.

You selected C; This is correct!
Reason: When a patient falls into the water or becomes panicked when in the water, he or she begins to swallow large amounts of water. Even a small amount of water near the larynx can cause a spasm, which closes off the ai

Which of the following conditions would MOST likely cause flushed skin?
� A:Low blood pressure
� B:Blood loss
� C:Hypothermia
� D:Exposure to heat

You selected D; This is correct!
Reason:Whenever the body temperature rises (ie, heat exposure, fever), the peripheral blood vessels dilate, which draws warm blood to the skin and gives it a flushed (red) appearance. Blood loss, shock, low blood pressure

Which of the following mechanisms cause respiratory and circulatory collapse during anaphylactic shock?
� A:Bronchodilation and vasodilation
� B:Bronchoconstriction and vasoconstriction
� C:Bronchoconstriction and vasodilation
� D:Bronchodilation and vaso

You selected C; This is correct!
Reason:During anaphylaxis, histamines released from the immune system cause two negative effects that result in shock (hypoperfusion): vasodilation, which causes the blood pressure to fall and bronchoconstriction, which im

A 50-year-old female complains of severe pain to the right lower quadrant of her abdomen. You should:
� A:palpate the left upper quadrant of her abdomen first.
� B:keep her supine with her legs fully extended.
� C:quickly palpate that area first to assess

You selected A; This is correct!
Reason:When assessing a patient with abdominal pain, determine where the pain is most severe (in this case, the right lower quadrant) and then palpate the quadrant furthest from that area first (in this case, the left uppe

A 30-year-old man with a history of schizophrenia cut his wrists and is bleeding profusely. He is confused, combative, and has slurred speech. With the assistance of law enforcement personnel, you and your partner physically restrain him in order to provi

You selected D; This is correct!
Reason:An adult with decision-making capacity (ie, a mentally competent adult) has the legal right to refuse medical treatment, even if that treatment involves lifesaving care. In psychiatric cases, however, a court of law

After removing a patient from the water, your assessment reveals that he is not breathing and is continuously regurgitating large amounts of water. You should:
� A:perform abdominal thrusts to remove the water.
� B:begin rescue breathing after he stops re

You selected C; This is correct!
Reason:In cases where a patient is not breathing and is regurgitating (passively vomiting) secretions at the same time, you must address both issues. This is accomplished most effectively by suctioning for 15 seconds and t

Approximately 5 minutes after being stung by a bee, a 21-year-old male develops hives and begins experiencing difficulty breathing. When you arrive at the scene, you note that his level of consciousness is decreased, his breathing is labored, and wheezing

You selected C; This is correct!
Reason: The patient is in anaphylactic shock and needs epinephrine immediately. Epinephrine is given to patients with anaphylactic shock because it constricts the blood vessels and dilates the bronchioles, thus improving p

Which of the following signs is LEAST suggestive of a diabetic emergency?
� A:Fruity breath odor
� B:Bradycardia
� C:Combativeness
� D:Tachycardia

You selected B; This is correct!
Reason:Bradycardia is not commonly associated with either hyperglycemia or hypoglycemia. Tachycardia and combativeness can occur in patients with hyperglycemia or hypoglycemia. A fruity breath odor is noted exclusively in

A 46-year-old man presents with generalized weakness and shortness of breath after he was bitten on the leg by a rattlesnake. His blood pressure is 106/58 mm Hg and his pulse rate is 112 beats/min. In addition to supplemental oxygen, further treatment for

The correct answer is C;
Reason:Care for a patient with a bite from a pit viper (rattlesnake, copperhead, water moccasin) includes keeping the patient calm, administering supplemental oxygen, splinting the affected part, and keeping it below the level of

Which of the following is a later sign of hepatitis?
� A:Fever and vomiting
� B:Jaundice
� C:Loss of appetite
� D:Fatigue

You selected B; This is correct!
Reason: Early signs and symptoms of viral hepatitis include loss of appetite (anorexia), vomiting, fever, fatigue, and muscle and joint pain. Jaundice (yellow sclera and skin) and right upper quadrant abdominal pain are no

A 60-year-old male complains of a tearing sensation in his abdomen. He tells you the pain began suddenly and feels like someone is sticking a knife into his abdomen. He is conscious and alert with a blood pressure of 148/88 mm Hg, a pulse of 120 beats/min

The correct answer is D;
Reason:Given the onset and nature of the patient's pain (eg, sudden onset, tearing sensation), you should suspect that he has an acute dissection of the abdominal aorta. Administer high-flow oxygen and transport him at once. Acute

Which of the following statements regarding the function of insulin is correct?
� A:It stimulates the liver to release glucose into the bloodstream.
� B:It facilitates the uptake of glucose from the bloodstream into the cell.
� C:It causes the pancreas to

You selected B; This is correct!
Reason:Insulin is a hormone produced by the beta cells in the Islets of Langerhans of the pancreas. It promotes the uptake of glucose from the bloodstream into the cells where it is used in the production of energy. Glucag

A 72-year-old woman is found unresponsive in her poorly ventilated home. Her skin is flushed, hot, and dry, and her respirations are rapid and shallow. She is wearing a medical alert bracelet that states she is a diabetic and is allergic to sulfa drugs. Y

You selected C; This is correct!
Reason: The patient's signs and symptoms are consistent with classic heat stroke. Unlike exertional heat stroke, which usually affects otherwise healthy people who exert themselves in the heat for long periods of time, cla

A 36-year-old male, who is a known diabetic, presents with severe weakness, diaphoresis, and tachycardia. He is conscious, but confused. His blood pressure is 110/58 mm Hg, pulse is 120 beats/min and weak, and respirations are 24 breaths/min. The glucomet

You selected D; This is correct!
Reason:The patient's signs and symptoms indicate hypoglycemia. When you are in doubt as to a patient's blood glucose level, you should err on the side of caution and give sugar; this is what medical control will likely ord

After moving a hypothermic patient to a warmer area, your primary focus should be to:
� A:prevent further body heat loss.
� B:provide rapid rewarming.
� C:assess his or her body temperature.
� D:give warm, humidified oxygen.

You selected A; This is correct!
Reason:After moving a hypothermic patient to a warmer area, your primary focus should be to prevent further body heat loss. Remove wet or damp clothing and cover the patient with blankets (passive rewarming). It is especia

Rapid transport of a patient who ingested a large dose of Tylenol is important because:
� A:it only takes a small dose of Tylenol to cause cardiopulmonary arrest.
� B:liver failure usually occurs within 6 hours following a Tylenol overdose.
� C:an antidot

You selected C; This is correct!
Reason:Acetaminophen (APAP), the active ingredient in Tylenol, is a safe drug if taken as directed. However, ingestion of more than 140 mg/kg in an adult can cause liver failure and death. Symptoms of APAP overdose do not

Which of the following structures is responsible for regulating body temperature?
� A:Cerebrum
� B:Medulla oblongata
� C:Hypothalamus
� D:Cerebellum

You selected B; The correct answer is C;
Reason:The hypothalamus, which is located within the brainstem, regulates body temperature by acting as the body's thermostat. During a heat-related emergency, the hypothalamus can "reset" the body's normal tempera

A young male experienced a syncopal episode after working in the heat for several hours. He is conscious and alert; has cool, clammy skin; and complains of nausea and lightheadedness. You should:
� A:provide rapid cooling.
� B:give him cold water to drink

The correct answer is D;
Reason:The patient is experiencing heat exhaustion and should be transported to the hospital for evaluation, especially since he experienced a syncopal episode (fainting). Because he is nauseated, he should not be given anything t

Which of the following is a physiologic effect of epinephrine when used to treat anaphylactic shock?
� A:As a vasodilator, it increases the blood pressure.
� B:As an antihistamine, it blocks chemicals that cause the reaction.
� C:As a vasoconstrictor, it

You selected D; This is correct!
Reason:Epinephrine possesses dual effects. As a bronchodilator, it relaxes the smooth muscle of the bronchioles and improves the patient's breathing. As a vasoconstrictor, it constricts the blood vessels and increases the

A 73-year-old male presents with confusion; cool, pale, clammy skin; absent radial pulses; and a blood pressure of 70/40 mm Hg. The patient's wife tells you that he has had abdominal pain for a week and began vomiting a coffee-ground substance yesterday.

You selected B; This is correct!
Reason:The patient is likely bleeding from his gastrointestinal (GI) tract. Although this is a serious condition, it is not a condition you can treat; internal bleeding cannot be controlled in the field. You can, however,

The two MOST important steps in treating a patient with a contact poisoning are:
� A:determining when the exposure occurred and irrigating the patient's entire body with copious amounts of water.
� B:avoiding self-contamination and removing the irritating

You selected B; This is correct!
Reason:When caring for a patient with a contact (skin surface) exposure to a poison, the two most important steps in management are avoiding contaminating yourself and removing the substance from the patient's skin as rapi

A 28-year-old woman has severe lower quadrant abdominal pain. When assessing her abdomen, you should:
� A:encourage the patient to lie supine with her legs fully extended.
� B:ask her where the pain is located and palpate that area first.
� C:auscultate f

You selected D; This is correct!
Reason:Assessment of a patient's abdomen includes asking where the pain is located and then palpating that area last. Palpating the painful area first may interfere with the rest of your assessment because of the significa

You are caring for a conscious patient who you believe is having an acute ischemic stroke. After administering oxygen, your next priority should include:
� A:providing prompt transport for possible fibrinolytic therapy.
� B:administering 1 tube of oral gl

You selected A; This is correct!
Reason:Fibrinolytic (clot-buster) therapy is critical to a patient who is having a stroke if it is initiated within 3 hours after the onset of symptoms. It is critical to ask a family member or bystander when the patient w

You are called to the residence of a 39-year-old male with flu-like symptoms. The patient tells you that he has been feeling ill for the last 2 days. He has a fever, headache, and diarrhea, and asks that you take him to the hospital. His blood pressure is

You selected D; This is correct!
Reason:Although the patient is likely experiencing the flu, there are other diseases, some of which are communicable, that can cause similar symptoms. The patient is requesting EMS transport; failure to comply constitutes

In general, you should avoid rewarming a frostbitten body part in the field if:
� A:arrival at the emergency department will be delayed.
� B:the affected part could refreeze after rewarming.
� C:you are unable to obtain water that is at least 120�F.
� D:a

The correct answer is B;
Reason:Frostbitten body parts should not be rewarmed if there is a chance that they could refreeze after you have rewarmed them. If an extremity thaws and then refreezes, the amount of tissue and cellular damage may be worse than

When caring for a patient with an acute behavioral crisis, your primary concern should be:
� A:gathering all of the patient's medications.
� B:obtaining a complete past medical history.
� C:ensuring you and your partner's safety.
� D:providing safe transp

You selected C; This is correct!
Reason:When caring for a patient with a behavioral or emotional crisis, your primary concern should be for your own personal safety as well as your partner's safety. Your ultimate goal is to transport the patient to the ho

Unresponsiveness, shallow breathing, and constricted pupils are indicative of what type of drug overdose?
� A:Marijuana
� B:Amphetamine
� C:Narcotic
� D:Barbiturate

You selected C; This is correct!
Reason:Signs of a narcotic (opiate) overdose from drugs such as heroin, morphine (Astromorph, Duramorph), meperidine (Demerol), or codeine include altered mental status; slow, shallow breathing; pupillary constriction (mio

Propoxyphene (Darvon) is categorized as what type of drug?
� A:Narcotic
� B:Amphetamine
� C:Benzodiazepine
� D:Barbiturate

The correct answer is A;
Reason:Propoxyphene (Darvon) is in the narcotic (opiate) class of drugs. Other narcotics include heroin, morphine, codeine, and meperidine (Demerol). Phenobarbital is an example of a barbiturate. Drugs such as Valium, Klonopin, an

A 60-year-old man presents with a severe right-sided headache. He is conscious and alert. As you are assessing him, he tells you that the left side of his body feels weak. His blood pressure is 190/100 mm Hg, pulse is 88 beats/min, and respirations are 14

You selected D; This is correct!
Reason:Headaches are a common medical complaint and have numerous causes, most of which are not life-threatening. Common causes of a headache include migraine and cluster headaches, sinusitis, and muscle tension, among oth

Which of the following would MOST likely cause a rapid drop in a patient's blood glucose level?
� A:Mild exertion after eating a meal
� B:Forgetting to take prescribed insulin
� C:Taking too much prescribed insulin
� D:Eating a meal after taking insulin

You selected C; This is correct!
Reason:Compared to diabetic coma (hyperglycemic ketoacidosis, hyperglycemic crisis), insulin shock (hypoglycemia, hypoglycemic crisis) has a rapid onset. It is commonly caused when a patient accidentally takes too much pre

A 44-year-old woman was bitten on the ankle by an unidentified snake while working in her garden. She is conscious and alert, has stable vital signs, and denies shortness of breath. Her only complaint is a burning sensation at the wound site. Your assessm

You selected B; This is correct!
Reason:Given the fact that the snake was not identified, you should assume that it was poisonous. Furthermore, the presence of puncture wounds, burning, redness, and swelling are suggestive of envenomation. Therefore, you

A 34-year-old female complains of persistent fever, fatigue, and night sweats. During your assessment, you note that she has purple blotches on her arms and legs. She MOST likely has:
� A:HIV/AIDS.
� B:chickenpox.
� C:tuberculosis.
� D:viral hepatitis.

You selected A; This is correct!
Reason:Signs of human immunodeficiency virus (HIV) infection include persistent fever, weight loss, fatigue, a cough, and night sweats. The presence of purple skin blotches (malignant lesions called Kaposi's sarcoma) is co

The daughter of an elderly patient states that her mother is acting confused and talking incoherently. This nature of illness is MOST consistent with:
� A:diabetic complications.
� B:altered mental status.
� C:cardiac compromise.
� D:behavioral problems

You selected B; This is correct!
Reason:The nature of illness is a category in which you place the patient based on his or her chief complaint. A chief complaint of confusion and incoherent speech suggests that altered mental status is the nature of illne

Which of the following statements regarding heatstroke is correct?
� A:Heatstroke is caused by a hyperactive sweating mechanism.
� B:Not all patients experiencing heatstroke have dry skin.
� C:As core body temperature rises, the patient becomes more agita

You selected B; This is correct!
Reason: Heatstroke occurs when the body is exposed to more heat than it can eliminate and normal mechanisms for eliminating heat, such as sweating, are overwhelmed. The core body temperature then rises rapidly to the point

Law enforcement requests your assistance to evaluate a young female who they believe has overdosed. The patient is conscious, is displaying bizarre behavior, and is very restless. Her BP is 170/90 mm Hg and her heart rate is 130 beats/min. You see needle

You selected D; This is correct!
Reason:The needle tracks on the patient's arms indicate narcotic abuse, most likely heroin; however, her signs and symptoms are not consistent with those caused by a narcotic or any other central nervous system (CNS) depre

Which of the following signs and symptoms are MOST characteristic of hyperglycemic ketoacidosis?
� A:Warm, dry skin and a slow onset
� B:Warm, dry skin and a rapid onset
� C:Cool, clammy skin and a rapid onset
� D:Cool, clammy skin and a slow onset

You selected A; This is correct!
Reason:Hyperglycemic ketoacidosis (diabetic coma) is characterized by a dangerously high blood glucose level (hyperglycemia); slow onset; warm, dry skin (from dehydration); and Kussmaul's respirations, which are deep and r

Which of the following statements regarding lightning-related injuries is correct?
� A:Multiple fractures, including those of the cervical spine, are the most common cause of lightning-related deaths.
� B:The majority of patients who are struck by lightni

The correct answer is D;
Reason:The cardiovascular and nervous systems are most commonly injured during a lightning strike; therefore, respiratory or cardiac arrest is the most common cause of lightning-related deaths, especially if CPR is delayed. A ligh

You are transporting a 30-year-old man who is experiencing an emotional crisis. The patient does not speak when you ask him questions. How should you respond to his unwillingness to speak?
� A:Remain silent until the patient speaks to you.
� B:Do not spea

You selected A; This is correct!
Reason:When dealing with a psychiatric patient who is silent and unwilling to speak to you, do not fear the silence. The patient simply does not wish to speak. You should not press the issue, for doing so may upset the pat

An elderly man is found unresponsive in his kitchen. The patient's wife tells you that her husband has diabetes and that he took his insulin, but did not eat anything. You should suspect:
� A:diabetic coma.
� B:hypoglycemia.
� C:ketoacidosis.
� D:hypergly

The correct answer is B;
Reason:Several factors point to a field impression of hypoglycemia. First, the patient is known to have diabetes and second, he took his insulin but did not eat. Because insulin promotes the uptake of glucose into the cells, if th

You and your partner arrive at the home of a 60-year-old man with shortness of breath. As you enter the residence, you find the patient sitting in his recliner; he is in obvious respiratory distress. As you approach him, he becomes verbally abusive, stati

You selected B; This is correct!
Reason:There is an immediate threat to you and your partner's safety! The patient is verbally abusive and has a firearm within arm's reach. In this situation, you and your partner should slowly back away from the patient,

A 55-year-old woman with a history of diabetes is found unresponsive with rapid, shallow respirations. The patient's husband tells you that he does not know when his wife last took her insulin. Management of this patient should include:
� A:subcutaneous i

You selected D; This is correct!
Reason:Without knowing if and when the patient last took her insulin, it is difficult to determine if she is experiencing diabetic coma or insulin shock. Nonetheless, her rapid, shallow respirations�which are likely not pr

When treating an unresponsive man who was struck by lightning, you should:
� A:manually stabilize his head and open his airway.
� B:apply full spinal precautions before moving him.
� C:begin CPR at once if he is apneic and pulseless.
� D:ensure that you a

You selected D; This is correct!
Reason:Contrary to popular belief, lightning can (and does) strike in the same place twice. After lightning strikes, the ground remains electrically charged for a period of time; this increases the chance of a second strik

You are assessing a 40-year-old male with an apparent psychiatric crisis. According to the mans's wife, he has a history of depression and schizophrenia and takes Zoloft and Zyprexa. A police officer is present at the scene. The patient, who is obviously

You selected C; This is correct!
Reason:When caring for any patient with a behavioral emergency, your primary responsibility is to yourself, to stay safe. The patient in this scenario, although scared, is not violent. In cases such as this, you should be

Which of the following structures is NOT part of the endocrine system?
� A:Gallbladder
� B:Pancreas
� C:Thyroid
� D:Pituitary

You selected A; This is correct!
Reason:The gallbladder, which concentrates and stores bile, is not an endocrine organ; it is a digestive organ. Endocrine organs produce hormones, which regulate other body organs and systems. The thyroid regulates metabol

Assessment and treatment of a patient with severe abdominal pain should include:
� A:giving the patient small sips of water.
� B:placing the patient in a supine position.
� C:administering supplemental oxygen.
� D:vigorously palpating the abdomen.

You selected C; This is correct!
Reason:Patients with severe abdominal pain should be given supplemental oxygen (in a concentration sufficient to maintain an SpO2 of greater than 94%) and immediate transport. Most patients with abdominal pain prefer to li

You receive a call for a 54-year-old female who is having a seizure. When you arrive at the scene, the patient is actively seizing. Her husband tells you that she has a history of seizures, and that she has been seizing continuously for 20 minutes. It is

You selected D; This is correct!
Reason:
There are many causes of seizures, including noncompliance with prescribed anticonvulsant medications, hypoglycemia, stroke, head trauma, and poisoning, among others. Regardless of the underlying cause, it is most

When insulin levels in the blood remain high:
� A:a fruity odor can be detected on the patient's breath.
� B:glucose is rapidly taken out of the blood to fuel the cells.
� C:the patient urinates excessively and becomes dehydrated.
� D:the cells starve for

The correct answer is B;
Reason:Insulin is a hormone that promotes the cellular uptake of glucose from the bloodstream. If insulin levels remain high, such as when a diabetic inadvertently takes too much insulin, glucose is rapidly taken out of the blood

The bite of a brown recluse spider is characterized by:
� A:two small puncture marks, swelling, and delayed onset of pain.
� B:immediate pain, swelling at the site, and painful muscle spasms.
� C:rapid swelling within 30 minutes and a decline in mental st

You selected D; This is correct!
Reason:The venom of a brown recluse spider is cytotoxic; that is, it causes tissue and cellular damage. The bite itself is usually painless; however, the patient typically begins to complain of pain within a few hours. The

Which of the following is the MOST significant finding in a patient experiencing an allergic reaction?
� A:Widespread rash.
� B:Abdominal cramps.
� C:Headache
� D:Hoarseness

You selected D; This is correct!
Reason:A rash (urticaria), headache, abdominal cramps, and hoarseness are all signs of an allergic reaction. However, the presence of hoarseness is the most significant finding because it indicates upper airway swelling. O

You receive a call to a restaurant where a 34-year-old man is experiencing shortness of breath. When you arrive, you immediately note that the man has urticaria on his face and arms. He is conscious, but restless, and is in obvious respiratory distress. Y

You selected B; This is correct!
Reason:Initial management of a patient with a suspected allergic reaction is to ensure a patent airway and provide high-flow oxygen. Positive-pressure ventilation may be required if the patient is breathing inadequately. A

Your patient is a 75-year-old female who, according to her son, tripped on a throw rug and fell. The patient is conscious and alert, is sitting on the couch, and has a hematoma to her forehead. When you ask her what happened, her son interjects by saying,

You selected A; This is correct!
Reason:There are several indicators that suggest this patient's injury was intentionally inflicted: the son's interjection, her hesitance to answer your questions, and the fact that she keeps looking at her son. If possibl

Which of the following signs or symptoms is MOST suggestive of a systemic reaction following ingestion of a poison?
� A:Painful or difficult swallowing
� B:Burns around the mouth
� C:Tachycardia and hypotension
� D:Nausea and vomiting

You selected C; This is correct!
Reason:Once an ingested poison gets into the system, it can affect multiple organ systems. Signs that this is occurring include tachycardia or bradycardia, hypotension or hypertension, weakness, restlessness, and an altere

A patient with a core body temperature of 94�F would MOST likely present with:
� A:joint and muscle stiffness.
� B:shivering and pale skin.
� C:decreased muscle activity.
� D:an altered mental status.

The correct answer is B;
Reason:Mild hypothermia is defined as a core body temperature (CBT) between 93.2�F and 96.8�F (34�C and 36�C). Mildly hypothermic patients are usually alert and shivering in an attempt to generate heat through muscular activity. T

An 80-year-old woman has pain in the right upper quadrant of her abdomen and a yellow tinge to her skin. You should suspect dysfunction of the:
� A:gallbladder.
� B:liver.
� C:pancreas.
� D:spleen.

You selected B; This is correct!
Reason:Pain in the right upper quadrant and skin with a yellow tinge to it (jaundice) indicates a problem with the liver. Jaundice is the result of an excess level of bilirubin in the blood. Bilirubin, a yellow-colored sub

A man is experiencing a severe allergic reaction after being stung by a scorpion. He does not have his own epinephrine; however, his wife is allergic to bees and has a prescribed epinephrine auto-injector. You should:
� A:request an ALS unit to respond to

You selected B; This is correct!
Reason:If a patient does not have a prescribed epinephrine auto-injector and is experiencing a severe allergic reaction, you should administer supplemental oxygen, assist the patient's ventilations if needed, and transport

You are transporting a 35-year-old male who has a history of alcoholism. He stopped drinking 4 days ago and is now disoriented, diaphoretic, and tachycardic. You should be MOST concerned that he:
� A:may have a seizure.
� B:is severely dehydrated.
� C:wil

You selected A; This is correct!
Reason:A patient in alcohol withdrawal may experience frightening hallucinations, or delirium tremens (DTs), a syndrome characterized by restlessness, fever, disorientation, sweating, agitation, and even seizures. About 1

While triaging patients at a mass-casualty incident, you encounter a responsive middle-aged female with a respiratory rate of 26 breaths/min. What should you do next?
� A:Triage her as immediate (red tag)
� B:Administer high-flow oxygen at once
� C:Assess

You selected C; This is correct!
Reason: According to the Simple Triage And Rapid Treatment (START) method, if you encounter an adult patient with a respiratory rate that is less than 10 breaths/min or greater than 29 breaths/min, you should triage him or

The ultimate goal of any EMS quality improvement program is to:
� A:recognize all EMTs who demonstrate consistency in providing competent patient care.
� B:provide protocols to all EMTs and hold them accountable if protocols are not followed.
� C:deliver

You selected C; This is correct!
Reason: Providing continuing education to all personnel, recognizing those who consistently provide competent patient care, and holding all personnel accountable for adhering to the EMS protocols are all components of any

A set of regulations and ethical considerations that define the extent or limits of an EMT's job is called:
� A:the Medical Practices Act.
� B:confidentiality.
� C:a duty to act.
� D:scope of practice.

You selected D; This is correct!
Reason:The set of legal regulations and ethical considerations that define the job of the EMT is called the scope of practice. The scope of practice provides a clear delineation of the EMT's roles and responsibilities. Dut

The scene size-up includes all of the following components, EXCEPT:
� A:evaluating the mechanism of injury.
� B:determining if the scene is safe.
� C:assessing the need for assistance.
� D:donning personal protective gea

You selected A; The correct answer is D;
Reason: The components of the scene size-up include determining scene safety, assessing the mechanism of injury (MOI) or nature of illness (NOI), determining the number of patients, and requesting additional help i

You are called to transport a patient with terminal lung cancer from a skilled nursing facility to the emergency department for evaluation of possible pneumonia. As you are reviewing the transfer paperwork, you see that the patient has a valid "do not att

You selected B; This is correct!
Reason:A valid do not attempt resuscitation (DNAR) order gives you the legal authority not to initiate resuscitative efforts if the patient develops cardiac arrest. Because laws vary from state to state, you must be famili

Which of the following statements BEST describes a mass-casualty incident?
� A:The number of patients overwhelms your resources.
� B:More than three vehicles are involved in the incident.
� C:More than five patients are involved.
� D:At least half of the

You selected A; This is correct!
Reason:A mass-casualty incident (MCI) occurs any time the number of injured patients overwhelms your available resources. It is not necessarily defined by the number of patients, but rather your ability to effectively mana

While caring for a trauma patient, blood splashes into an EMT's eyes. This is an example of:
� A:infection.
� B:indirect contact.
� C:transmission.
� D:exposure.

You selected D; This is correct!
Reason: Exposure occurs when an individual comes in direct contact with blood or other bodily fluids. Examples of direct exposure include blood splashing into the eyes or mouth and an accidental stick with a contaminated n

Failure of the EMT to obtain consent from a responsive patient before taking his or her blood pressure may constitute:
� A:assault.
� B:negligence.
� C:abandonment.
� D:battery.

You selected D; This is correct!
Reason:Battery is defined as unlawfully touching another person without his or her consent. Obtaining consent from every responsive patient prior to rendering care is of paramount importance. Assault is defined as instilli

The EMT should avoid focusing all of his or her attention on a single critical patient during the triage process because:
� A:the patient will most likely die before he or she can be transported.
� B:all of his or her supplies will likely be depleted on t

You selected D; This is correct!
Reason: Triage is the process of rapidly assessing patients in order to determine their treatment priority. Focusing your attention on one patient during the triage process not only defeats the purpose of triage (to do the

You are cleaning the back of the ambulance after transporting a patient with major trauma. Which of the following contaminated items should NOT be placed in a plastic biohazard bag?
� A:Suction canister
� B:Plastic IV catheter
� C:Rigid suction catheter

You selected B; This is correct!
Reason:Contrary to popular belief, the end of a Teflon IV catheter (the plastic catheter without the needle) can cut you; it is sharp! Therefore, it should be placed in a puncture-proof sharps container, not a plastic bioh

At the scene of a mass-casualty incident, you notice a bystander who is emotionally upset. An appropriate action to take would be to:
� A:tell the bystander to leave the scene at once.
� B:assign the bystander a simple, non-patient-care task.
� C:have the

You selected B; This is correct!
Reason:One of the most effective ways to reduce stress in a bystander at the scene of a mass-casualty incident is to assign the bystander a task that is not related to patient care. This may involve assisting other bystand

Which of the following statements regarding the high-efficiency particulate air (HEPA) respirator is correct?
� A:A HEPA respirator is necessary only if the patient with suspected tuberculosis is coughing.
� B:Long sideburns or a beard will prevent the pr

You selected B; This is correct!
Reason:If you are caring for a patient with known or suspected tuberculosis (TB), regardless of whether the patient is coughing, you should place a surgical mask (or high-flow oxygen, if indicated) on the patient and a hig

You arrive at the scene of a young male who was stabbed when a burglar broke into his house. Law enforcement officers are present. The patient, who is unresponsive with several stab wounds to the chest, is lying in a narrow space between a couch and coffe

You selected D; This is correct!
Reason:After ensuring your own safety, your primary responsibility when functioning at a crime scene is to provide patient care. Optimally, you should help preserve the scene at the same time. In this situation, however, t

Which of the following scene size-up findings is LEAST suggestive of an unsafe environment?
� A:The sound of breaking glass as you approach a residence
� B:Screaming and yelling coming from inside a residence
� C:Liquid draining from a car that struck a t

You selected D; This is correct!
Reason: When approaching a residence, findings that would suggest an unsafe scene include, among other things, the sound of breaking glass, screaming and yelling, and an unusual silence. Liquid leaking from a wrecked autom

You and your partner have secured a trauma patient to a long backboard and are preparing to lift the backboard onto the stretcher. When doing so, you should:
� A:recall that most of the patient's weight is at the foot end of the backboard.
� B:ensure that

You selected B; This is correct!
Reason:Since more than half of the patient's weight is distributed to the head end of a backboard, you should always ensure that the strongest EMT is at that position. This will reduce the risk of injury to less strong per

Upon arriving at the scene of an overturned tanker truck, you see a clear liquid leaking from the rear of the tank. The driver is still in the vehicle and you can see that his face is covered with blood. You should:
� A:request fire department assistance

You selected A; This is correct!
Reason:Upon determining that you are dealing with a potential hazardous materials incident, you should immediately request specially trained personnel (eg, fire department, Haz-Mat team) if they are not already en route to

Which of the following situations is an example of abandonment?
� A:An EMT gives a verbal report to an emergency nurse.
� B:A paramedic transfers care to an advanced EMT.
� C:An EMT departs the scene after a paramedic arrives.
� D:An EMT transfers care of

You selected B; This is correct!
Reason:Abandonment occurs any time you disengage from a patient while he or she still requires care or you relinquish your responsibility of patient care to a provider of lesser training. If a paramedic transfers patient c

As soon as you begin transport of a patient to the hospital, you should:
� A:notify the receiving facility.
� B:conduct a detailed examination.
� C:contact medical control.
� D:advise dispatch of your status.

You selected D; This is correct!
Reason:Immediately upon departing the scene with a patient, you should first inform the dispatcher that you are en route to the hospital. Never leave the dispatcher in the dark, for it is the dispatcher's job to know what

Upon arriving at the scene of a multiple vehicle crash, you can see that at least two patients have been ejected from their vehicles. You should:
� A:immediately triage the two patients.
� B:gather all of the patients together.
� C:treat the most critical

You selected D; This is correct!
Reason:One ambulance and two EMTs can effectively care for only one critical patient or two non-critical patients. As soon as you determine that the patient count exceeds your capabilities, you should immediately call for

While en route to the scene of an injured person, dispatch advises you that law enforcement personnel are at the scene. This indicates that:
� A:the scene is safe for you to enter.
� B:a crime has been committed.
� C:the patient is critically injured.
� D

You selected D; This is correct!
Reason:The presence of law enforcement at the scene indicates, at a minimum, that the scene is potentially unsafe, otherwise they would not be there. While you are en route, you should make radio contact with the police of

When sizing up a motor-vehicle crash in which a small passenger car struck a bridge pillar and sustained severe damage, you should:
� A:disconnect the battery cables and then request heavy extrication tools.
� B:ensure that there are no hazards and then t

You selected B; This is correct!
Reason:After ensuring your own safety, you should attempt simple access to the patient, trying to get to him or her as simply and quickly as possible without using any tools or breaking any glass. It may be necessary to us

Which of the following patients has the highest priority at the scene of a mass-casualty incident?
� A:44-year-old unresponsive man with an open head injury and agonal gasps
� B:29-year-old man who is pulseless and apneic with an abdominal evisceration

You selected C; This is correct!
Reason:The goal of triage is to provide the greatest good for the greatest number of people. Relative to the other patients, who are either in respiratory or cardiac arrest or have injuries incompatible with life, the woma

Which of the following practices will provide you with the highest degree of safety when responding to an emergency call?
� A:Always requesting fire department assistance
� B:Routinely using the lights and siren
� C:Wearing your seatbelt and shoulder harn

You selected C; This is correct!
Reason:Consistent safety practices, such as wearing your seatbelt and shoulder harness, driving with due regard for others, and avoiding excessive speed, will afford you the highest degree of safety when responding to an e

Which of the following situations would necessitate treatment using implied consent?
� A:A 17-year-old pregnant woman with an isolated extremity injury
� B:A 65-year-old man who is confused and suspected of having a severe stroke
� C:A 25-year-old man who

You selected B; This is correct!
Reason:A patient may be treated under the law of implied consent, also called the emergency doctrine, any time he or she is unresponsive or otherwise lacks decision-making capacity (ie, confused, under the influence of dru

You are called to a residence for a woman in cardiac arrest. Shortly after starting CPR, the patient's husband presents you with an unsigned document that states "do not attempt resuscitation." You should:
� A:contact medical control prior to continuing a

The correct answer is C;
Reason:Do not attempt resuscitate (DNAR) orders are particularly challenging for EMS providers. When presented with documentation, especially if it does not appear to be valid (in this case, an unsigned document), you should err o

In which of the following situations is an emergency move of a patient from his or her wrecked vehicle clearly indicated?
� A:The patient appears unresponsive and a high-power line is lying across the hood.
� B:The patient has an altered mental status; di

The correct answer is C;
Reason:An emergency move is indicated if you or the patient's life is in immediate danger. Gas leaking from the vehicle and a fire in the engine compartment are clear indicators that you and the patient's lives are in imminent dan

Proper body mechanics when lifting and moving a patient include:
� A:keeping the weight as close to you as possible.
� B:twisting at the waist when moving around a corner.
� C:using the muscles of your lower back to lift.
� D:maintaining a slight curvatur

You selected A; This is correct!
Reason:General guidelines for safe lifting and moving include keeping the weight as close to your body as possible; keeping your back in a straight, locked-in position; using the muscles of your thighs to lift; and avoidin

Your partner, a veteran EMT of 20 years, has been showing up late to work with increasing frequency over the last several shifts. When he arrives, he is in a bad mood and is clearly not interested in being at work. His behavior is MOST consistent with:

You selected B; This is correct!
Reason:Your partner's behavior is consistent with burnout. Burnout is a condition of chronic fatigue, irritability, and frustration that results from mounting stress over time. Although burnout typically manifests after ye

When requesting medical direction for a patient who was involved in a major motor-vehicle accident, you should do all of the following, EXCEPT:
� A:describe the severity of damage to the patient's vehicle.
� B:use radio codes to describe the situation.

The correct answer is B;
Reason:When giving a report to medical control or requesting medical direction, you should avoid the use of radio codes, such as "10-50." The physician may not be familiar with such codes. There is clearly less risk of confusion i

While assisting a paramedic in starting an IV on a patient, you are inadvertently stuck by the contaminated needle while attempting to place it in the sharps container. You should:
� A:notify your supervisor and request an HIV test.
� B:hold the paramedic

You selected D; This is correct!
Reason:If you receive an exposure while providing patient care, you should notify your supervisor or designated infection control officer and seek medical care as soon as possible. However, this should be done after you ha

You are called to a local state park where a hiker fell from a cliff into a thick, wooded area with rough terrain. It is cold and foggy and a thunderstorm is approaching. You will MOST likely move the patient to the ambulance by:
� A:bringing the ambulanc

The correct answer is C;
Reason:The basket stretcher, also called a Stokes basket, is ideal for moving patients across rough terrain. Because the patient fell from a cliff, he will require spinal motion restriction precautions. Secure him to a long backbo

You receive a call at 3:00 a.m. for a patient who is slumped over the steering wheel of his car, which is parked on the shoulder of the road. Your unit and a police officer arrive at the scene at the same time. You should:
� A:stay in your unit until the

You selected A; This is correct!
Reason:Unfortunately, it is not uncommon for people to fake illness or injury with the intent of harming responding personnel. In this situation, you should utilize the safety resource at the scene: the police officer. You

As an EMT, your primary responsibility is to:
� A:keep yourself as safe as possible.
� B:transport all patients to the hospital.
� C:ensure the safety of your partner.
� D:provide competent patient care.

You selected A; This is correct!
Reason:As an EMT, your primary responsibility is to yourself. An injured or dead EMT is of no use to a patient. After ensuring the safety of yourself, your crew, and any bystanders, patient care should be initiated.

Which of the following situations presents the greatest risk for suicide?
� A:A woman who quit her job for one that pays a lot more
� B:A man who was recently diagnosed with stage 4 lung cancer
� C:An EMT who saved a drowning child and receives no media a

You selected B; This is correct!
Reason:Any patient with a significant, most often negative, life change is at risk for suicide. Common catalysts to suicide include chronic depression, the loss of a loved one or a job, relationship problems, financial dif

A 30-year-old woman crashed her car into a tree at a high rate of speed. She is conscious and alert and has stable vital signs. She has some small lacerations and abrasions to her arms and face, but no obviously life-threatening injuries. As you are loadi

You selected C; This is correct!
Reason:The consequences of refusal should be explained to any patient who refuses EMS treatment and/or transport. After establishing that the patient can legally refuse treatment and transport (eg, he or she is of legal ag

A 70-year-old man with a history of emphysema and congestive heart failure is in cardiac arrest. His wife tells you that he collapsed about 5 minutes before your arrival. Your partner begins one-rescuer CPR as you prepare the AED. As you are applying the

You selected D; This is correct!
Reason:When faced with a situation in which a family member does not wish for you to attempt resuscitation of a loved one, you should inquire about the presence of a living will or out-of-hospital do not attempt resuscitat

How does a unified incident command system differ from a single incident command system?
� A:In a unified incident command system, a single incident commander is identified and will function as such, regardless of the type of incident
� B:In a unified inc

The correct answer is B;
Reason:Regardless of the type of incident command system (ICS) used, a single incident commander (IC) must be in charge. In a unified incident command system, plans are drawn up in advance by all cooperating agencies that assume a

After arriving at a mass-casualty incident where other ambulances are already present, you should notify the dispatcher and then:
� A:initiate care for the most critically injured patients.
� B:obtain information from the fire service commander.
� C:repea

You selected D; This is correct!
Reason:Once you arrive at the scene of a mass-casualty incident where an incident command system has already been established, you should report to the staging area, the area designated for all incoming ambulances and reso

You are caring for a 6-year-old child with a swollen, painful deformity to the left forearm. As you communicate with the parents of this child, you should:
� A:make sure that they remain aware of what you are doing.
� B:tell them that the child will be tr

You selected A; This is correct!
Reason:When caring for any patient, it is important to keep both the patient and family aware of what you are doing. You should avoid medical terminology whenever possible because most laypeople will not understand what yo

You arrive at the scene where a man fell approximately 40 feet and landed on his head. He is unresponsive, has agonal gasps, and a weak carotid pulse. Further assessment reveals an open head injury with exposed brain matter. Upon identifying this patient

The correct answer is B;
Reason:It is unlikely that the patient will survive his injury; however, he is still breathing, albeit very poorly, and has a weak carotid pulse. Therefore, you should begin immediate and aggressive treatment and transport him pro

According to the United States Department of Transportation (USDOT), minimum staffing for a basic life support ambulance includes:
� A:at least two EMTs in the patient compartment.
� B:a minimum of two EMTs in the ambulance.
� C:an EMT who functions as th

You selected D; This is correct!
Reason:The United States Department of Transportation (USDOT) requires at least one EMT in the patient compartment of a basic life support (BLS) ambulance. Although the driver does not have to be an EMT, it is preferable.

When is it MOST appropriate to complete your patient care report for a critically ill or injured patient?
� A:Promptly after the primary assessment
� B:As soon as all patient care activities are completed
� C:After the ambulance has been restocked at the

You selected B; This is correct!
Reason:Patient care activities, especially when the patient's condition is critical, take priority over the completion of your patient care report (PCR). Once all patient care activities have been completed, you can comple

While functioning at a large-scale terrorist incident, it is important for the EMT to:
� A:use triage and base patient care on available resources.
� B:avoid placing any casualty in a "delayed" treatment status.
� C:identify the person or persons responsi

You selected A; This is correct!
Reason:During a terrorist incident, the basic foundations of triage and patient care remain the same; however, the treatment can and will vary. Terrorist incidents can produce a single casualty, hundreds of casualties, or

As you step out of the ambulance at the scene of a nighttime motor vehicle crash on the highway, your MOST immediate concern should be:
� A:the presence of oncoming traffic.
� B:whether the car will catch on fire.
� C:rapid assessment of all injured patie

You selected A; This is correct!
Reason:Nighttime traffic crashes, especially those that occur on a highway, pose a significant risk to the safety of the EMT. Therefore, immediately upon exiting the ambulance, the concern for oncoming traffic should be at

Which of the following is an example of the EMT providing care based on standing orders?
� A:Following assessment of a patient with a terminal illness, the EMT notifies the patient's personal physician to determine whether he or she requires any special c

The correct answer is B;
Reason:Standing orders define certain treatment interventions that the EMT is authorized to perform prior to contacting medical control. For example, the EMT knows that a cardiac arrest patient requires CPR, cardiac rhythm analysi

The role of triage officer at a mass-casualty incident should be assumed by the:
� A:first EMS provider who is willing to perform the task.
� B:EMS medical director via telephone communication.
� C:most knowledgeable EMS provider.
� D:EMS provider with th

The correct answer is C;
Reason:The role of triage officer should be assumed by the most knowledgeable EMS provider at the scene. Knowledge and experience will enable this person to most effectively manage the triage process. Just because a person has bee

You are giving a presentation to a group of laypeople on the importance of calling EMS immediately for cardiac arrest patients. What point should you emphasize the MOST?
� A:Laypeople are incapable of providing adequate CPR.
� B:CPR and defibrillation are

The correct answer is B;
Reason:Early high-quality CPR and defibrillation are the most crucial initial treatments to provide to a patient in cardiac arrest. Adequately performed CPR can keep the heart and brain oxygenated, thus increasing the chance of de

Which of the following vaccinations is NOT currently recommended by the Centers for Disease Control and Prevention (CDC)?
� A:Tetanus, diphtheria, and pertussis
� B:Measles, mumps, rubella
� C:Hepatitis B
� D:Smallpox

The correct answer is D;
Reason:Immunization requirements are usually set by the state department of health, but generally follow recommendations made by the Centers for Disease Control and Prevention (CDC). Recommended immunizations for health care provi

Which of the following creates a secure loop at the working end of a rope, which can be used to attach the end of the rope to a fixed object or a piece of equipment?
� A:Half hitch
� B:Figure eight knot
� C:Clove hitch
� D:Figure eight on a bite

The correct answer is D;
Reason:Although EMTs infrequently perform special rescue operations (ie, urban search and rescue [USAR]), they should have a basic working knowledge of the different ropes and knots used for rescue purposes. The figure eight on a

Most crashes involving ambulances occur:
� A:at intersections.
� B:at stop lights.
� C:at stop signs.
� D:on the highway.

You selected A; This is correct!
Reason:Intersection crashes are the most common and usually the most serious type of collision in which ambulances are involved. When approaching an intersection, you should come to a complete stop, look in both directions

While you are inside a crashed vehicle assessing a patient who is entrapped, the rescue team should be:
� A:actively extricating the patient using whichever extrication method they deem necessary.
� B:preparing for a simple extrication process since you w

You selected D; This is correct!
Reason:Your ability to access the patient, with or without difficulty, does not indicate the extent of entrapment or method of extrication necessary. The EMT's job is to assess and treat the patient; the rescue team's job

You are the first ambulance to arrive at the scene of a motor-vehicle crash. As you approach the scene, you see three patients, two who have been ejected from their vehicles and the other who is still in his vehicle. You should:
� A:notify the local traum

You selected C; This is correct!
Reason:One of the most important aspects of the scene size-up is to determine the need for additional resources. A single EMS unit and two EMTs cannot effectively manage three patients, especially if they are critically in

When called to the scene of a structural fire to stand by in case any injuries occur at the scene, you should:
� A:depart the scene and return to service after the fire has been completely extinguished.
� B:enter the structure with the firefighters so you

You selected D; This is correct!
Reason:When standing by at the scene of a structural fire, you must ensure that the ambulance is parked at a safe distance; this will not only keep you safe from the fire itself, but also minimize your risk of injury if th

When calling your radio report to the receiving hospital, you should:
� A:include the patient's name.
� B:break your report into 60-second increments.
� C:only give your report to a physician.
� D:be brief, concise, and factual.

You selected D; This is correct!
Reason:A radio report should be brief, concise, and factual. It should include the patient's age and sex, his or her chief complaint, associated assessment findings, vital signs, treatment that you provided, and the patien

Following a call in which a 6-week-old infant in cardiac arrest did not survive, your partner is exhibiting significant anxiety and irritability. How can you MOST effectively help him?
� A:Report his behavior to the medical director.
� B:Tell him that he

You selected D; This is correct!
Reason:Your partner is clearly having difficulty coming to terms with this call's bad outcome. As his partner, you can be most effective during this time by simply listening and allowing him to voice his feelings. Bad feel

Medical control has ordered you to administer one tube of oral glucose to a hypoglycemic patient. Immediately after receiving this order, you should:
� A:ask medical control to repeat the order word for word.
� B:repeat the order back to medical control w

You selected B; This is correct!
Reason:Immediately after receiving an order from medical control, you should repeat the order back to medical control word for word. This will ensure that you heard correctly and understand the order to be carried out. If

Your actions at the scene of a critically injured patient who was shot during a robbery should include:
� A:caring for the patient while manipulating the scene minimally.
� B:performing a primary assessment only.
� C:providing care when the police authori

The correct answer is A;
Reason:After ensuring your own safety, your priority while functioning at a crime scene is to provide care to the patient. However, you should make a reasonable effort to avoid manipulating the scene in order to preserve potential

A conscious and alert 92-year-old woman with chest pain is refusing EMS treatment and transport to the hospital. Her family insists that you transport her. This situation is MOST appropriately managed by:
� A:transporting the patient as the family wishes.

You selected D; This is correct!
Reason:Just because the patient is 92 years old does not mean that she does not have decision-making capacity and cannot make an informed decision. In cases where any patient refuses care, after determining that the patien

The technique of rapid extrication from a vehicle involves:
� A:manually stabilizing the head, applying a cervical collar, and removing the patient from the vehicle onto a long backboard.
� B:grabbing the patient by his or her clothing, protecting his or

You selected A; This is correct!
Reason:The rapid extrication technique is indicated if the patient has life-threatening injuries and/or is in need of treatment that requires a supine position. It is performed by manually stabilizing the patient's head (a

A man armed with a shotgun has taken two people hostage and has shot one of them. Upon arriving at the scene, you should:
� A:be sure and turn up your portable radio loud enough so that you can remain aware of the entire situation.
� B:have the incident c

You selected B; This is correct!
Reason:Upon arriving at the scene of a tactical situation, such as a hostage situation, the presence of a sniper, or any exchange of gunfire, you should report to the incident commander, who will take you to a shielded, sa

All of the following are considered key components at the scene of a mass-casualty incident, EXCEPT:
� A:an adequately staffed treatment area.
� B:a supply area near the treatment area.
� C:an on-scene emergency physician.
� D:an on-site communication sys

You selected C; This is correct!
Reason:On-site communications, an adequately staffed treatment area, and a supply area near the treatment area are some of the key components at a mass-casualty incident. Although the physician medical director is ultimate

The MOST effective means of preventing the spread of disease is:
� A:wearing a mask with all patients.
� B:effective handwashing.
� C:wearing gloves with all patients.
� D:up-to-date immunizations.

You selected B; This is correct!
Reason:According to the Centers for Disease Control and Prevention (CDC), the most effective way of preventing the spread of disease is to frequently and effectively wash your hands, especially in between patients. The reg

The information that would be of LEAST pertinence when educating the public on injury prevention is:
� A:how to provide rescue breathing.
� B:teaching children to wear bicycle helmets.
� C:the proper usage of child safety seats.
� D:building a childproof

You selected A; This is correct!
Reason:The goal of an injury prevention program is just that, prevention. If rescue breathing is needed in a situation, the injury has already occurred. As EMS providers, we are consequence managers. Additionally, we have

While en route back to your station from the emergency department, you discover that you forgot to include vital patient information on the front of your patient care report (PCR). Having left a copy of your PCR with the emergency department staff, you sh

You selected C; This is correct!
Reason:Once you leave a copy of your patient care report (PCR) with the hospital staff, you should not add information to the front of the report. If you discover that you forgot to include vital patient information, you s

You have completed your patient care report and left a copy at the hospital when you realize that you forgot to document a pertinent finding on the front of the report. You should:
� A:take no action and report the event to your supervisor.
� B:attach an

You selected B; This is correct!
Reason:If you discover that you forgot to include pertinent information on your patient care report (PCR) after leaving a copy at the hospital, you should write the information on a separate addendum and attach it to the o

While treating a patient in cardiac arrest, you turn the AED on and attach the pads to the patient. However, when the AED begins to analyze the patient's cardiac rhythm, it signals "low battery" and then shuts off. The patient subsequently died. Which of

You selected A; This is correct!
Reason:The most common errors that occur with the AED are the result of operator error (not equipment failure), usually because no one made sure that the batteries were fully charged when checking the ambulance at the star

When providing care to multiple patients at the scene of a mass-casualty incident, your goal should remain focused on:
� A:transporting patients to the hospital.
� B:keeping all bystanders at a safe distance.
� C:initiating CPR for those in cardiac arrest

You selected A; This is correct!
Reason:At the scene a mass-casualty incident, you will be faced with many challenges, including ensuring your safety, extrication, triage, and patient care. In the midst of all of these activities, however, you must never

You arrive at the scene of an 80-year-old woman who is weak and lightheaded. Her son, who called 911, is present and asks you to transport his mother to the hospital. You should:
� A:assess the woman and determine if she wishes to be treated and transport

You selected A; This is correct!
Reason:You must obtain consent from any mentally competent adult patient prior to initiating treatment. Just because the patient is 80 years old does not mean that she does not have decision-making capacity. And just becau

Which of the following actions demonstrates an EMT's knowledge of crime scene preservation?
� A:Carefully cutting through the hole in a patient's clothing that was made by a large caliber firearm
� B:Requesting approval from law enforcement before control

You selected C; This is correct!
Reason:After ensuring your own safety, your priority when caring for a patient at a crime scene is to do just that, care for the patient. If you need to move a piece of furniture to gain access to a critically injured pati

While caring for an emotionally disturbed 30-year-old male, he suddenly becomes violent and needs to be physically restrained. During the restraint procedure, you should:
� A:approach the patient from the front and converge on him quickly.
� B:maintain co

The correct answer is B;
Reason:Physical restraint may be necessary if a patient is a threat to him- or herself or others. If it becomes necessary to physically restrain a violent patient, there are certain, specific guidelines that must be followed. Use

The safest emergency vehicle operator is one who:
� A:has a positive attitude.
� B:is physically fit.
� C:drives with lights and siren.
� D:drives with due regard.

You selected D; This is correct!
Reason:One of the most important attributes of a safe emergency vehicle operator is the ability to drive with due regard for others. This means that the operator must be aware of others around him or her and to keep their

The immobilization device MOST appropriate to use for a patient with multiple injuries and unstable vital signs is the:
� A:long spine board immobilization device.
� B:short spine board immobilization device.
� C:vest-style immobilization device.
� D:scoo

You selected A; This is correct!
Reason:When caring for a critically injured patient with multiple injuries, the patient's entire body should be immobilized. This is most quickly and effectively accomplished using a long spine board. Vest-style devices or

When driving in emergency mode on a multilane highway, the emergency vehicle operator should keep to the:
� A:right shoulder so that traffic flow is not disrupted.
� B:extreme right lane so motorists can yield to the left.
� C:extreme left lane so motoris

You selected C; This is correct!
Reason:When traveling on a highway with more than one lane, the emergency vehicle operator should remain in the extreme left-hand (fast) lane. This allows other motorists to yield to the right as they see you approach. A m

When using the power lift to lift a stretcher, you should:
� A:ensure that you lift with your palms facing up.
� B:place your hands palms down on the stretcher.
� C:maintain a slight inward curve to your back.
� D:bend at the waist and keep your back stra

You selected A; This is correct!
Reason:In order to achieve the best grip and to avoid injury to your wrists, you should lift a stretcher, backboard, or other carrying device with your palms facing up. Do not bend at the waist; bend at the knees and keep

A 52-year-old woman crashed her minivan into a tree. She is pinned at the legs by the steering wheel and is semiconscious. After gaining access to the patient, you should:
� A:have the fire department disentangle the patient and quickly remove her from th

You selected B; This is correct!
Reason:Unless there is an immediate threat of fire, explosion, or other danger, you should perform a primary assessment and begin any live-saving care as soon as you have gained access to the patient. If you wait to do thi

Which of the following is the MOST practical method of standard precautions when treating multiple patients during a mass-casualty incident?
� A:Asking each patient you treat if he or she has a communicable disease
� B:Placing clean gloves over soiled glo

You selected D; This is correct!
Reason:Although the most effective means of preventing the spread of disease involves thorough hand-washing, this is not practical at the scene of a mass-casualty incident; you usually do not have the time or facilities to

Upon arriving at a scene in which a tanker truck overturned and is spilling an unknown liquid on the ground, you should:
� A:quickly identify the material.
� B:turn off your warning lights.
� C:park upwind from the scene.
� D:stay downhill from the scene.

You selected C; This is correct!
Reason:At the scene of a potential or actual hazardous materials incident, you should park the ambulance in an area that is both upwind and uphill from the incident. However, you must be prepared to quickly relocate if the

Who has ultimate authority for all issues regarding patient care at the scene of a mass-casualty incident?
� A:Incident commander
� B:Most experienced EMT
� C:EMS medical director
� D:Treatment officer

You selected C; This is correct!
Reason:The incident commander (IC) is responsible for all logistical and operational aspects of a mass-casualty incident (MCI), such as designating section officers and working in conjunction with other agencies (eg, polic

Following an apparent terrorist attack, numerous patients present with shortness of breath and persistent coughing. A green haze is noted in the area in which the patients are located. Which of the following agents should you suspect they were exposed to?

The correct answer is D;
Reason:The patient's signs and symptoms are indicative of a pulmonary (choking) agent, specifically chlorine (CL). Chlorine (CL) was the first chemical agent ever used in warfare. It has a distinct odor of bleach and creates a gre

Upon arriving at the scene of a crash involving a large truck, you immediately note the presence of an orange placard on the side of the tank that the truck is pulling. This indicates that the vehicle is carrying a/an ______________ agent.
� A:explosive

The correct answer is A;
Reason:The color of a warning placard indicates the general classification of agent being carried, while the United Nations (UN) number in the center of the placard indicates the exact agent being carried. For example, a red placa

After assisting a patient with her epinephrine auto-injector, you should:
� A:place the device in a puncture-proof container.
� B:place the device in a red biohazard bag.
� C:give it to the patient to have it refilled.
� D:replace the cover and place it i

You selected A; This is correct!
Reason:After any device is used that has the potential for causing an accidental needle stick or is otherwise contaminated, it should be placed in a puncture-proof container, which usually is red and has a biohazard logo o

In most states, the EMT is required to report which of the following occurrences?
� A:Animal bite
� B:Motor vehicle crash
� C:Injury to a minor
� D:Drug overdose

You selected A; This is correct!
Reason:Although each state may have slightly differing reporting laws, most require the EMT to report cases such as child or elderly abuse, sexual assault, animal bites, and injury that occurs during the commission of a cr

A patient presents with severe bradycardia, hypersalivation, vomiting, and excessive tearing. Which of the following agents would MOST likely cause his signs and symptoms?
� A:Phosgene
� B:Chlorine
� C:Soman
� D:Anthrax

The correct answer is C;
Reason:Nerve agents (eg, V agent [VX], sarin [GB], soman [GD], tabun [GA]) are among the most deadly chemicals developed. Designed to kill large numbers of people with small quantities, nerve agents can cause cardiac arrest within

Which of the following statements regarding the use of an escort vehicle when en route to an emergency call is correct?
� A:An escort vehicle will allow you to arrive at the scene quicker.
� B:An escort vehicle should be used only if you are unfamiliar wi

The correct answer is B;
Reason:Generally, escort vehicles should not be used when responding to an emergency scene. The biggest danger of using an escort occurs at intersections, which is where most ambulance crashes occur. Drivers may yield to the escor

You are dispatched to a call for an unresponsive patient. Which of the following is the MOST important information that you should initially obtain from the dispatcher?
� A:The exact location of the patient
� B:Whether or not the patient is breathing
� C:

You selected A; This is correct!
Reason:You should attempt to obtain as much patient information from the dispatcher as you can. However, in order to help the patient, you must know his or her location; therefore, this is the most important initial questi

Which of the following is considered minimum personal protective equipment (PPE) when suctioning an unresponsive patient's airway?
� A:Gloves and a mask
� B:Gloves, gown, and eye protection
� C:Gloves, head cover, and eye protection
� D:Gloves and full fa

You selected D; This is correct!
Reason:You should wear gloves when assessing or treating any patient. The level of personal protective equipment (PPE) used beyond gloves is dictated by the risk and type of exposure. When managing a patient's airway (eg,

You are called to treat a 55-year-old man who is experiencing difficulty breathing. After making contact with your patient, he extends his arm out to allow you to take his blood pressure. This is an example of:
� A:implied consent.
� B:formal consent.
� C

The correct answer is C;
Reason:Actual consent, also referred to as expressed consent, is when the patient asks for your help outright. This may also include nonverbal gestures, such as extending the arm to you to allow you to take the blood pressure. Inf

Which of the following components is NOT part of an injury or illness prevention program?
� A:Teaching new parents how to properly install a child safety seat
� B:Inspecting the home of young parents for child safety locks
� C:Educating teenage students o

You selected D; This is correct!
Reason:The operative word in injury and illness prevention is "prevention." An injury or illness prevention program should focus on actions that prevent illness or injury. These include, but are not limited to, teaching ne

When the incident command system is activated at the scene, you should expect to:
� A:report back to your section officer in between assignments.
� B:receive instructions and then function independently.
� C:be assigned one responsibility for the duration

You selected A; This is correct!
Reason:The incident command system (ICS) was established in order to maximize effective operations at the scene and maximize the number of lives saved. Deviation from the ICS jeopardizes lives and increases the risk of los

While caring for an injured patient, you remove blood-soaked clothing in order to treat his injuries. You should dispose of the clothing by:
� A:leaving it at the hospital.
� B:leaving it at the scene.
� C:placing it in a regular trash can.
� D:placing it

You selected D; This is correct!
Reason: The appropriate method for disposing of soiled clothing or any other "nonsharp" contaminated item is to place it in a red biohazard bag. The biohazard insignia as well as the red color alerts others that the items

During the triage process, which of the following injuries or conditions would classify a patient as a high priority?
� A:Partial-thickness burns with no respiratory difficulty
� B:A large avulsion to the arm and an altered mental status
� C:Pulselessness

The correct answer is B;
Reason:During triage, patients with an altered mental status, who are in shock, or who have problems with airway, breathing, or circulation, are potentially salvageable and are given immediate priority. Patients who are pulseless

The primary clinical feature associated with exposure to a vesicant agent is:
� A:vomiting blood.
� B:muscle twitching.
� C:tachycardia.
� D:skin blistering.

You selected D; This is correct!
Reason:The primary route of exposure of blister agents, or vesicants, is the skin. If vesicants are left on the skin or clothing long enough, they produce vapors that can enter the respiratory tract. Vesicants cause burn-l

You arrive at the scene of a traffic accident in which multiple vehicles are involved. You see at least two patients who are lying on the road and are not moving. You should:
� A:begin triaging the patients.
� B:request additional ambulances.
� C:begin im

You selected B; This is correct!
Reason:As soon as you determine that there are more patients than you and your partner can effectively manage, you should immediately request additional help. Waiting until you are overwhelmed with critically injured patie

Proper guidelines for safe reaching include all of the following, EXCEPT:
� A:keeping your back in a locked-in position.
� B:avoiding twisting of your back.
� C:reaching no more than 30" in front of your body.
� D:avoiding hyperextension of your back.

You selected C; This is correct!
Reason:Safe reaching practices are critical to the prevention of a back injury. When reaching, you should keep your back in a locked-in position. You should avoid twisting or hyperextending your back, and should reach no m

Which position is MOST appropriate for a mother in labor with a prolapsed umbilical cord?
� A:Supine with legs elevated
� B:Left lateral recumbent
� C:Supine with hips elevated
� D:Left side with legs elevated

You selected C; This is correct!
Reason:When the umbilical cord is prolapsed, the infant typically slides down the birth canal and rests on top of the cord, shutting off its own oxygen supply. Placing the mother supine with her hips elevated will cause th

Which of the following statements regarding two-rescuer child CPR is correct?
� A:A compression to ventilation ratio of 15:2 should be delivered without pauses in compressions to deliver ventilations
� B:The chest should be compressed with one hand and a

You selected C; This is correct!
Reason:When performing two-rescuer CPR on a child (1 year of age to the onset of puberty [12 to 14 years of age]), the chest should be compressed with one or two hands (depending on the size of the child), and a compressio

Your assessment of a mother in labor reveals that a fetal limb is protruding from the vagina. Management of this situation should include:
� A:positioning the mother with her hips elevated, administering high-flow oxygen, and providing transport.
� B:appl

The correct answer is A;
Reason:Limb presentations represent a dire emergency for the newborn and do not spontaneously deliver in the field. You should position the mother in a manner so that her hips are elevated in an attempt to slide the infant slightl

Following delivery of a newborn and placenta, you note that the mother has moderate vaginal bleeding. The mother is conscious and alert and her vital signs are stable. Treatment for her should include:
� A:treating her for shock and providing rapid transp

You selected C; This is correct!
Reason: Postpartum bleeding is most effectively controlled by massaging the fundus (top) of the uterus. Uterine massage stimulates the pituitary gland to secrete a hormone called oxytocin, which constricts the blood vessel

You are assessing a 5-year-old boy with major trauma. His blood pressure is 70/40 mm Hg and his pulse rate is 140 beats/min. and weak. The child's blood pressure:
� A:is appropriate based on his age.
� B:suggests increased intracranial pressure.
� C:indic

The correct answer is C;
Reason:The low normal systolic blood pressure (SBP) for a child between 1 and 10 years of age is calculated by multiplying his or her age (in years) by 2 and adding 70. Using this formula, the low normal SBP for a 5-year-old child

The MAIN reason why small children should ride in the backseat of a vehicle is because:
� A:their legs are highly prone to injury from striking the dashboard.
� B:they can experience severe injury or death if the airbag deploys.
� C:they are much less lik

You selected B; This is correct!
Reason:Children under 12 years of age should ride in the backseat of a vehicle, preferably in the middle, and restrained in a device that is appropriate for their size. Merely placing the child in the backseat does not red

A 4-year-old boy ingested an unknown quantity of drain cleaner. He is alert, has a patent airway, and has adequate breathing. You should:
� A:administer 1 g/kg of activated charcoal.
� B:give 15 mL of ipecac and contact medical control.
� C:contact poison

You selected C; This is correct!
Reason:Once you determine that a poisoning has occurred, and have identified the poison, you should contact the poison control center at once: (800) 222-1222. Give the patient high-flow oxygen or assist his or her ventilat

Following the initial steps of resuscitation, a newborn remains apneic and cyanotic. You should:
� A:start CPR if the heart rate is less than 80 beats/min.
� B:immediately resuction its mouth and nose.
� C:gently flick the soles of its feet for up to 60 s

The correct answer is D;
Reason:The initial steps of newborn resuscitation, which are performed on all newborns following delivery, include drying, warming, positioning, suctioning, and tactile stimulation. If the newborn remains apneic after the initial

Which of the following is a more reliable indicator of perfusion in children than it is in adults?
� A:Heart rate
� B:Capillary refill
� C:Blood pressure
� D:Respiratory rate

You selected B; This is correct!
Reason:In children younger than 6 years of age, capillary refill time (CRT) serves as an excellent indicator of perfusion; it assesses oxygen delivery to the capillaries. As a person gets older, however, CRT becomes less r

Which artery should you palpate when assessing for a pulse in an unresponsive 6-month-old patient?
� A:Radial
� B:Brachial
� C:Femoral
� D:Carotid

You selected B; This is correct!
Reason:You should assess the brachial pulse in infants younger than 1 year of age. The carotid or femoral pulse can be assessed in children older than 1 year of age. A carotid pulse is difficult to locate in infants becaus

You are caring for a 6-year-old child with a possible fractured arm and have reason to believe that the child was abused. How should you manage this situation?
� A:Call the police and have the parents arrested.
� B:Transport the child to the hospital rega

You selected C; This is correct!
Reason:The responding EMT must handle cases of suspected child abuse with great care. You must never accuse the parents or caregiver of abuse. If you are wrong, you could be held liable for slander. Actions that would sugg

The preferred method for inserting an oropharyngeal airway in a small child is to:
� A:insert the airway as you would in an adult, but use an airway that is one size smaller than you would normally use.
� B:open the airway with the tongue-jaw lift maneuve

You selected C; This is correct!
Reason:Keeping in mind that a child's tongue is proportionately large, the preferred method for inserting an oropharyngeal (oral) airway is to use a tongue blade to depress the tongue and slide the airway straight in, with

A 3-year-old female presents with respiratory distress. She is conscious, crying, and clinging to her mother. She has mild intercostal retractions and an oxygen saturation of 93%. The MOST effective way of delivering oxygen to her involves:
� A:ventilatio

You selected C; This is correct!
Reason:Do not assume that a child will simply allow you to administer oxygen to him or her as you would to an adult. The child in this scenario, who is in respiratory distress and is mildly hypoxemic (SpO2 of 93%), should

The MOST important initial steps of assessing and managing a newborn include:
� A:drying and warming the infant and obtaining an APGAR score.
� B:suctioning the airway and obtaining a heart rate.
� C:clearing the airway and keeping the infant warm.
� D:ke

You selected C; This is correct!
Reason:In the initial steps of assessing and managing the newborn, the most important aspects include clearing the airway of amniotic fluid and making sure that the baby stays warm. The APGAR score should not be relied on

A 3-year-old child has a sudden onset of respiratory distress. The mother denies any recent illnesses or fever. You should suspect:
� A:lower respiratory infection.
� B:foreign body airway obstruction.
� C:croup.
� D:epiglottitis.

You selected B; This is correct!
Reason:You should suspect a foreign body airway obstruction in any child who presents with an acute onset of respiratory distress in the absence of fever. Croup, epiglottitis, and lower airway infections (ie, bronchiolitis

During transport of a woman in labor, the patient tells you that she feels the urge to push. You assess her and see the top of the baby's head bulging from the vagina. You should:
� A:ask the mother to take short, quick breaths until you arrive at the hos

You selected C; This is correct!
Reason:If, during transport, the mother begins to deliver the infant, your first action should be to advise your partner to stop the ambulance and assist you with the delivery. Delivery of a baby should never be attempted

If a woman is having her first child, the first stage of labor:
� A:is usually the longest and lasts an average of 16 hours.
� B:generally does not allow time for you to transport.
� C:is shorter than in women who have had other children.
� D:is typically

You selected A; This is correct!
Reason:There are three stages of labor: dilation of the cervix, delivery of the baby, and delivery of the placenta. The first stage begins with the onset of contractions and ends when the cervix is fully dilated. Since ass

A 3-year-old boy is found to be in cardiopulmonary arrest. As you begin one-rescuer CPR, your partner prepares the AED. The appropriate compression to ventilation ratio for this child is:
� A:3:1
� B:30:2
� C:5:1
� D:15:2

You selected B; This is correct!
Reason:A universal compression to ventilation ratio of 30:2 is used for all one-rescuer CPR (adult, child, and infant), with the exception of the newborn. A compression to ventilation ratio of 3:1 is used for newborns (one

You are dispatched to a residence for a 4-year-old female who is sick. Your assessment reveals that she has increased work of breathing and is making a high-pitched sound during inhalation. Her mother tells you that she has been running a high fever for t

You selected C; This is correct!
Reason:The child is clearly experiencing respiratory distress and probably has croup (laryngotracheobronchitis), a viral upper airway infection. The presence of stridor (high-pitched sound heard during inhalation) indicate

Which of the following injuries is MOST indicative of child abuse?
� A:Multiple bruises to the shins
� B:Small laceration to the chin
� C:Burned hand with splash marks
� D:Bruising to the upper back

You selected D; This is correct!
Reason:In order to detect child abuse, you must be familiar with injury locations and patterns consistent with an accident versus those that were intentionally inflicted. It is common for children to trip, fall, and strike

Which of the following statements regarding crowning is correct?
� A:It is safe to transport the patient during crowning if the hospital is close.
� B:Gentle pressure should be applied to the baby's head during crowning.
� C:Crowning always occurs immedia

You selected B; This is correct!
Reason:Crowning occurs when the baby's head is visible at the vaginal opening; it is an obvious sign of delivery in progress. When crowning is observed, you should apply gentle pressure to the infant's head to prevent an e

You receive a call for a 3-year-old girl with respiratory distress. When you enter her residence, you see the mother holding the child, who does not acknowledge your presence. This finding indicates that the child:
� A:is reacting normally for her age.

correct answer is B;
Reason:A 3-year-old child typically is very attentive to his or her surroundings, especially when a stranger enters the environment. The fact that this child does not acknowledge your presence is an abnormal sign and indicates signifi

Immediately upon delivery of a newborn's head, you should:
� A:suction the nose.
� B:suction the mouth.
� C:dry the face.
� D:cover the eyes.

You selected B; This is correct!
Reason:As soon as the newborn's head has delivered, you should first suction the mouth, then the nose. As the infant is forced through the birth canal, the thoracic cavity is squeezed, which causes the infant to expel amni

You arrive at a residence shortly after a 4-year-old boy experienced an apparent febrile seizure. The child is alert and crying. His skin is flushed, hot, and moist. His mother tells you that the seizure lasted about 2 minutes. You should:
� A:begin rapid

The correct answer is D;
Reason:Febrile seizures are common in children between the ages of 6 months and 6 years; they occur when the child's body temperature suddenly rises or when an already febrile child experiences an acute fever spike. Treatment for

Which of the following assessment parameters is a more reliable indicator of perfusion in infants than adults?
� A:Level of orientation
� B:Pulse quality
� C:Blood pressure
� D:Capillary refill

You selected D; This is correct!
Reason:Capillary refill time (CRT) is a reliable indicator of perfusion in children less than 6 years of age. When the capillary bed (eg, fingernail, forehead) is blanched, blood should return to the area in less than 2 se

Upon delivery of a baby's head, you see that the umbilical cord is wrapped around its neck. Initial treatment for this condition should include:
� A:clamping and cutting the umbilical cord.
� B:keeping the cord moist and providing rapid transport.
� C:try

You selected C; This is correct!
Reason:If you can see the umbilical cord wrapped around the newborn's neck (nuchal cord) when the head delivers, you should gently attempt to slide the cord from around the neck. If this is unsuccessful, you should clamp a

After clearing the airway of a newborn who is not in distress, it is MOST important for you to:
� A:obtain an APGAR score.
� B:apply blow-by oxygen.
� C:keep the newborn warm.
� D:clamp and cut the cord.

You selected C; This is correct!
Reason:After ensuring a patent airway (ie, suctioning and positioning), it is extremely important to keep the newborn warm. Newborns cannot maintain body temperature very well and hypothermia can develop very quickly. Blow

After an advanced airway device has been inserted in a 6-month-old infant in cardiopulmonary arrest, you should deliver ventilations at a rate of:
� A:6 to 8 breaths/min.
� B:10 to 12 breaths/min.
� C:8 to 10 breaths/min.
� D:12 to 20 breaths/min.

The correct answer is C;
Reason:After an advanced airway device (eg, ET tube, multilumen airway, supraglottic airway) has been inserted during cardiac arrest, ventilate the patient at a rate of 8 to 10 breaths/min (one breath every 6 to 8 seconds). This v

A 9-year-old girl was struck by a car while she was crossing the street. Your assessment reveals a large contusion over the left upper quadrant of her abdomen and signs of shock. Which of the following organs has MOST likely been injured?
� A:Kidney
� B:L

You selected D; This is correct!
Reason:Abdominal trauma commonly occurs in children as the result of motor vehicle versus pedestrian accidents. The contusions over the left upper quadrant and the signs of shock suggest significant injury to the spleen. T

A 7-year-old child has an altered mental status, high fever, and a generalized rash. You perform your assessment and administer supplemental oxygen. En route to the hospital, you should be MOST alert for:
� A:respiratory distress.
� B:combativeness.
� C:h

The correct answer is D;
Reason:High fever and an alerted mental status indicate sepsis (severe infection). A generalized rash should alert you to the possibility of meningitis�a condition caused by infection and inflammation of the meninges that protect

Supplemental oxygen via the blow-by technique is MOST appropriate for a child who presents with respiratory difficulty and:
� A:is agitated, tachycardic, and clinging to his or her parent.
� B:has a heart rate of 70 beats/min and signs of physical exhaust

You selected A; This is correct!
Reason:If a child presents with respiratory difficulty, the method of oxygen delivery depends on his or her mental status, respiratory effort, and heart rate. A child with respiratory distress has an increased work of brea

After the baby's head delivers, it is usually tilted:
� A:anteriorly, with the chin up.
� B:with the face up.
� C:posteriorly, to one side.
� D:posteriorly, face down.

The correct answer is C;
Reason:As the baby's head begins to deliver, it is usually in a posterior, face down position. After the head delivers completely, however, it usually tilts to the side in preparation for delivery of the shoulders. Remember to che

A 6-year-old boy complains of pain to the right lower quadrant of his abdomen. Assessment of this child's abdomen should include:
� A:palpating the right lower quadrant first.
� B:auscultating bowel sounds for 2 minutes.
� C:avoiding palpation of the abdo

You selected D; This is correct!
Reason: When assessing the abdomen of any patient, you should determine the location of the pain and palpate that area last. Begin by palpating the abdomen furthest away from the area of pain; in this case, the left upper

Treatment for a responsive 4-year-old child with a mild airway obstruction, who has respiratory distress, a strong cough, and normal skin color, includes:
� A:oxygen, back slaps, and transport.
� B:assisted ventilations, back slaps, and transport.
� C:sub

The correct answer is D;
Reason:If a child (1 year of age to the onset of puberty [12 to 14 years of age]) with a mild airway obstruction is alert and has adequate air movement (ie, a strong cough, normal skin color), you should offer oxygen, avoid agitat

When is it MOST appropriate to clamp and cut the umbilical cord?
� A:As soon as the cord has stopped pulsating
� B:Immediately following delivery of the newborn
� C:Before the newborn has taken its first breath
� D:After the placenta has completely delive

You selected A; This is correct!
Reason: Generally, it is safe to clamp and cut the umbilical cord once it has stopped pulsating and the baby is breathing adequately. When blood flow through the umbilical cord ceases, it will stop pulsating; this indicate

After attaching the AED to a 7-year-old child in cardiac arrest, you push the analyze button and receive a shock advised message. After delivering the shock, you should:
� A:reanalyze the cardiac rhythm.
� B:immediately perform CPR.
� C:assess for a carot

You selected B; This is correct!
Reason: After the AED delivers a shock, you should immediately begin or resume CPR, starting with chest compressions. Perform CPR for 2 minutes and then reanalyze the child's cardiac rhythm. If the AED states that a shock

Prevention of cardiac arrest in infants and small children should focus primarily on:
� A:providing immediate transport.
� B:avoiding upsetting the child.
� C:ensuring adequate ventilation.
� D:keeping the child warm.

You selected C; This is correct!
Reason:The most common cause of cardiac arrest in infants and children is failure of the respiratory system. Their hearts generally are healthy, and they rarely go into ventricular fibrillation (V-Fib). The key to preventi

A 29-year-old woman, who is 38 weeks pregnant, presents with heavy vaginal bleeding, a blood pressure of 70/50 mm Hg, and a heart rate of 130 beats/min. She is pale and diaphoretic, and denies abdominal cramping or pain. Her signs and symptoms are MOST co

The correct answer is A;
Reason:Of the conditions listed, placenta previa would be the least likely to present with abdominal pain, although some patients may have pain or cramping. Placenta previa is a condition in which the placenta develops over and co

Which of the following parameters is the LEAST reliable when assessing the perfusion status of a 2-year-old child?
� A:Skin color and temperature
� B:Presence of peripheral pulses
� C:Systolic blood pressure
� D:Capillary refill time

You selected C; This is correct!
Reason:You should never rely on the systolic blood pressure when assessing the perfusion status of anyone. More reliable parameters include assessing peripheral pulses, capillary refill time (most reliable in children youn

You are responding to a call for a 2-year-old child who fell from a second-story window. With the mechanism of injury and the age of the patient in mind, you should suspect that the primary injury occurred to the child's:
� A:head.
� B:lower extremities.

You selected A; This is correct!
Reason:Because a child's head is proportionately larger than the rest of the body when compared to an adult, the head commonly is the primary site of injury. This is especially true in fall-related injuries, in which gravi

Which of the following is a sign of an altered mental status in a small child?
� A:Consistent eye contact with the EMT.
� B:Inattention to the EMT's presence.
� C:Recognition of the parents.
� D:Fear of the EMT's presence.

You selected B; This is correct!
Reason:Typically, a small child will fear the presence of a stranger in his or her environment and will maintain constant eye contact with the stranger; therefore, inattentiveness to your presence should alert you to the p

Which of the following statements regarding pediatric anatomy is correct?
� A:Relative to the overall size of the airway, a child's epiglottis is smaller.
� B:The child's trachea is more rigid and less prone to collapse.
� C:Airway obstruction is common i

You selected D; This is correct!
Reason:Compared to adults, infants and small children have a proportionately larger head, specifically the occiput (back of the head). Therefore, when positioning an infant or a child's airway, padding in between the shoul

When you begin to assess a woman in labor, she states that her contractions are occurring every 4 to 5 minutes and lasting approximately 30 seconds each. Which of the following questions would be MOST appropriate to ask next?
� A:Have you had regular pren

The correct answer is B;
Reason:When assessing a patient in labor, the first question you should ask is how far along in the pregnancy she is. If she is at less than 37 weeks gestation (37 to 42 weeks is term), you should prepare for possible resuscitatio

You and your partner are performing CPR on an infant with suspected sudden infant death syndrome (SIDS). An important aspect in dealing with such cases is:
� A:remembering that most infants with SIDS can be successfully resuscitated.
� B:focusing all of y

You selected C; This is correct!
Reason:When managing an infant with suspected sudden infant death syndrome (SIDS), you will be faced with three tasks: assessment and management of the infant, communicating with and providing emotional support to the fami

Upon assessing a newborn immediately after delivery, you note that the infant is breathing spontaneously and has a heart rate of 80 beats/min. What is the MOST appropriate initial management for this newborn?
� A:Start chest compressions and contact medic

The correct answer is D;
Reason:Positive-pressure ventilations are indicated in the newborn if he or she is apneic or has gasping respirations, if the heart rate is less than 100 beats/min, or if central cyanosis persists despite the delivery of blow-by o

The appropriate technique for performing two-rescuer CPR on a 4-year-old child includes:
� A:15 compressions to 2 ventilations, compressing the sternum with the heel of your hand, and ventilating until visible chest rise occurs.
� B:30 compressions to 2 v

The correct answer is A;
Reason:When performing two-rescuer CPR on an infant (less than 1 year of age) or a child (1 year of age to the onset of puberty [12 to 14 years of age]), use a compression to ventilation ratio of 15:2. Compress the chest one-third

Appropriate treatment for an 18-year-old woman with severe vaginal bleeding may include all of the following, EXCEPT:
� A:covering the vagina with a trauma dressing.
� B:high concentrations of oxygen.
� C:placing sterile dressings into the vagina.
� D:kee

You selected C; This is correct!
Reason: The source of bleeding from the vagina cannot be directly controlled in the field. You should never pack or place any dressings directly into the vagina as this increases the risk of infection; furthermore, these d

A 34-year-old woman, who is 36 weeks pregnant, is having a seizure. After you protect her airway and ensure adequate ventilation, you should transport her:
� A:in a semisitting position.
� B:in the prone position
� C:on her left side.
� D:in the supine po

You selected C; This is correct!
Reason:Initial care for any patient who is seizing�pregnant or otherwise�involves ensuring a patent airway, adequate ventilation, and administering high-flow oxygen. If the patient is breathing inadequately, ventilation as

Which of the following statements regarding the length-based resuscitation tape measure is correct?
� A:The red end of the tape measure is placed at the heel of the child's foot.
� B:The resuscitation tape estimates a child's age based on his or her heigh

The correct answer is D;
Reason:There are a number of ways to identify the appropriate size equipment for a pediatric patient; however, the length-based resuscitation tape measure is perhaps the most accurate. Pediatricians generally agree that length (he

Your assessment of a 5-year-old child reveals that he is unresponsive with a respiratory rate of 8 breaths/min and a heart rate of 50 beats/min. Treatment for this child should include:
� A:high-flow oxygen via nonrebreathing mask and rapid transport.
� B

The correct answer is C;
Reason:A heart rate less than 60 beats/min in an infant or child�especially when accompanied by signs of poor perfusion and inadequate breathing�should be treated with positive-pressure ventilation, chest compressions, and rapid t

Which of the following techniques represents the MOST appropriate method of opening the airway of an infant with no suspected neck injury?
� A:Perform the technique as you would for an older child or adult.
� B:Lift up the chin and hyperextend the neck.

You selected C; This is correct!
Reason:Opening the airway in infants and small children involves keeping the head in a neutral or slightly extended position. Because the occipital region (back of the head) of the skull is proportionately larger in infant

Following delivery of a newborn, the 21-year-old mother is experiencing mild vaginal bleeding. You note that her heart rate has increased from 90 to 120 beats/min and she is diaphoretic. In addition to administering high-flow oxygen, treatment should incl

You selected C; This is correct!
Reason:Blood loss of up to 500 mL within the first 24 hours after delivery is considered normal and usually is well tolerated by the mother. However, any bleeding, regardless of the severity, with accompanying signs of sho

Which of the following signs is MOST indicative of inadequate breathing in an infant?
� A:Heart rate of 130 beats/min
� B:Sunken fontanelles
� C:Expiratory grunting
� D:Abdominal breathing

You selected C; This is correct!
Reason:Expiratory grunting in an infant or a child with a respiratory problem is an ominous sign; it indicates impending respiratory arrest. Grunting represents the child's attempt to maintain oxygen reserve in the lungs.

In contrast to the contractions associated with true labor, Braxton-Hicks contractions:
� A:consistently become stronger and are not alleviated by changing position.
� B:may be intensified by activity and are accompanied by a pink discharge.
� C:do not in

You selected C; This is correct!
Reason:During pregnancy, the mother may experience false labor, or Braxton-Hicks contractions, in which there are contractions but they do not represent true labor. Unlike true labor contractions, Braxton-Hicks contraction

A 5-year-old boy was struck by a car when he ran out into the street. When you arrive at the scene and approach the child, you see him lying supine approximately 15 feet from the car. Based on the child's age and mechanism of injury, which of the followin

The correct answer is D;
Reason:Children are smaller than adults; therefore, when they are injured by the same mechanism of injury as an adult, the location of their injuries may differ from those of an adult. For example, when an adult is struck by a veh

Upon delivery of the baby's head, you note that the umbilical cord is wrapped around its neck. You should:
� A:immediately clamp and cut the cord and continue the delivery.
� B:give the mother high-flow oxygen and transport her on her side.
� C:keep the c

You selected D; This is correct!
Reason:If the umbilical cord is wrapped around the baby's neck (nuchal cord), the EMT should make one attempt to gently remove the cord from around its neck. If this is unsuccessful, clamp and cut the cord and continue wit

You have just delivered a baby girl. Your assessment of the newborn reveals that she has a patent airway, is breathing adequately, and has a heart rate of 130 beats/min. Her face and trunk are pink, but her hands and feet are cyanotic. You have clamped an

You selected D; This is correct!
Reason: The newborn is stable and does not require care beyond providing thermal management and monitoring. Oxygen is indicated for the newborn if it has central cyanosis (cyanosis to the face and trunk), and should be del

The function of the uterus is to:
� A:house the fetus as it grows for 40 weeks.
� B:provide oxygen and other nutrients to the fetus.
� C:dilate and expel the baby from the cervix.
� D:provide a cushion and protect the fetus from infection.

You selected A; This is correct!
Reason:The uterus is a muscular organ where the fetus grows for 37 to 42 weeks (average of 40 weeks). It is responsible for contracting during labor, which in conjunction with dilation of the cervix (the opening of the ute

The 5-minute Apgar assessment of a newborn reveals a heart rate of 130 beats/min, cyanosis to the hands and feet, and rapid respirations. The infant cries when you flick the soles of its feet and resists attempts to straighten its legs. These findings equ

The correct answer is D;
Reason: The Apgar score, which is obtained at 1 and 5 minutes after birth (and every 5 minutes thereafter), assigns numbers (0, 1, or 2) to the following five areas: Appearance, Pulse, Grimace, Activity, and Respirations. A score

The transition phase of the pediatric assessment process would be the LEAST appropriate if:
� A:the child is unstable and needs rapid transport.
� B:you determine that the child's condition is stable.
� C:a parent is available to help keep the child calm.

The correct answer is A;
Reason:If the child's condition does not require immediate transport, the transition phase can allow the infant or child to become familiar with you and your equipment. This will help to alleviate the child's anxiety, allowing you

Of the following, the MOST detrimental effect of gastric distention in infants and children is:
� A:less effective chest compressions.
� B:acute rupture of the diaphragm.
� C:tracheal rupture.
� D:decreased ventilatory volume.

You selected D; This is correct!
Reason:Gastric distention can be lethal if not detected and managed appropriately in any patient, especially infants and children. As air insufflates the stomach, the diaphragm is pushed into the thoracic cavity, which dec

You are called to a local park for a 7-year-old boy with respiratory distress. During your assessment, you find that the patient is wheezing and has widespread hives and facial edema. What should you suspect has occurred?
� A:Poison oak exposure
� B:Acute

You selected C; This is correct!
Reason: Wheezing, hives, and edema are hallmark findings of an allergic reaction. In this case, the patient is having a severe reaction. Although wheezing occurs in patients with asthma, hives and facial edema do not. Whee

A newborn is considered to be premature if it:
� A:is born before 37 weeks gestation.
� B:weighs less than 6.5 pounds.
� C:is born to a heroin-addicted mother.
� D:has meconium in or around its mouth.

You selected A; This is correct!
Reason: A term gestation is between 37 and 42 weeks. A premature newborn is one that is born before 37 weeks gestation or weighs less than 5.5 lb (2.5 kg). Compared to women who do not abuse drugs, smoke, or drink alcohol

During your assessment of a woman in labor, you see the baby's arm protruding from the vagina. The mother tells you that she needs to push. You should:
� A:cover the arm with a sterile towel and transport immediately.
� B:gently push the protruding arm ba

You selected A; This is correct!
Reason: On rare occasions, the presenting part of the fetus is neither the head nor buttocks, but a single arm or leg. This is called a limb presentation. You cannot successfully deliver such a presentation in the field. T

In order to maintain neutral alignment of an 18-month-old child's airway, you should:
� A:pad in between the shoulder blades.
� B:hyperextend the head.
� C:ensure that the head is slightly flexed.
� D:place a rolled towel behind the head.

You selected A; This is correct!
Reason:Infants and small children have proportionately large heads, specifically the occiput (back of the head). Therefore, it is often necessary to place padding in between the scapulae (shoulder blades) in order to ensur

Which of the following is an abnormal finding?
� A:Respiratory rate of 26 breaths/min in a 2-year-old child
� B:Systolic BP of 100 mm Hg in a 10-year-old child
� C:Rapid, irregular breathing in a newly born infant
� D:Heart rate of 80 beats/min in a 3-mon

The correct answer is D;
Reason:An infant between 1 month and 1 year of age typically has a heart rate between 100 and 160 beats/min; a heart rate less than 100 beats/min in this age group is considered to be bradycardic. Newborn infants normally have irr

A 4-year-old girl fell from a third-story window and landed on her head. She is semiconscious with slow, irregular breathing and is bleeding from her mouth and nose. You should:
� A:suction her oropharynx, open her airway with the jaw-thrust maneuver, ins

The correct answer is B;
Reason:In any semi- or unconscious patient with a head injury, you should manually stabilize the head and open the airway with the jaw-thrust maneuver. If there are any secretions in the mouth, suction the oropharynx. If possible,

You are dispatched to a residence for a child having a seizure. When you arrive at the scene, the 4-year-old child's grandfather tells you that he has had several full body seizures over the past 20 minutes, but never woke up in between the seizures. The

The correct answer is C;
Reason:Status epilepticus is defined as a prolonged (greater than 20 minutes) seizure or multiple seizures without a return of consciousness in between seizures. A febrile seizure is caused by an abrupt rise in body temperature, u

A sudden onset of respiratory distress in a 5-year-old child with no fever is MOST likely the result of:
� A:a foreign body airway obstruction.
� B:inflammation of the upper airway.
� C:infection of the lower airways.
� D:a progressive upper airway infect

You selected A; This is correct!
Reason:Children with no fever who have a sudden onset of respiratory distress should be treated for a foreign body airway obstruction. If the child is able to cough, cry, or speak, he or she is moving adequate air and has

You arrive at the scene shortly after a 3-year-old female experienced a seizure. The child, who is being held by her mother, is conscious and crying. The mother tells you that her daughter has been ill recently and has a temperature of 102.5�F. What is th

You selected D; This is correct!
Reason:As evidenced by her recent illness and fever (102.5�F), this child has likely experienced a febrile seizure. Appropriate treatment for the child following a febrile seizure involves ensuring a patent airway, adminis

A prolapsed umbilical cord is dangerous because the:
� A:cord might pull the placenta from the uterine wall during delivery.
� B:baby's head may compress the cord, cutting off its supply of oxygen.
� C:mother may die of hypoxia due to compromised placenta

You selected B; This is correct!
Reason:A prolapsed umbilical cord, a condition in which a portion of the umbilical cord delivers before the baby, is a dangerous condition; the baby's head may compress the cord, cutting off its own supply of oxygen. There

You are administering oxygen at 15 L/min to a patient with respiratory distress. If you are using a D cylinder (cylinder constant, 0.16), which reads 1,500 psi, how long will it take before you need to replace the oxygen cylinder?
� A:18 minutes
� B:9 min

The correct answer is C;
Reason:
The length of time you can use an oxygen cylinder depends on the type of cylinder you are using, the pressure in the cylinder, and the oxygen flow rate. A D cylinder is a small oxygen cylinder that is usually carried in th

The pain associated with acute aortic dissection:
� A:is typically described as a stabbing or tearing sensation.
� B:typically comes on gradually and progressively worsens.
� C:is usually preceded by nausea, sweating, and weakness.
� D:originates in the e

You selected A; This is correct!
Reason:
Aortic dissection occurs when the inner layers of the aorta become separated, allowing blood to flow between the layers at high pressure. This separation of layers significantly weakens the aortic wall, making it p

A 33-year-old female presents with acute respiratory distress. She is conscious but anxious, and tells you that she has a history of asthma. She took two puffs of her albuterol inhaler prior to your arrival, but states that it did not help. Her oxygen sat

You selected B; This is correct!
Reason:
Despite two albuterol treatments, the patient is still experiencing respiratory distress. Furthermore, the presence of wheezing indicates continued bronchospasm. After administering high-flow oxygen via a nonrebrea

Which of the following BEST describes the mechanism of injury?
� A:The product of mass, force of gravity, and height
� B:The energy of an object in motion
� C:Your concern for potentially serious injuries
� D:The way in which traumatic injuries occur

You selected D; This is correct!
Reason:
The mechanism of injury (MOI) is the way in which traumatic injuries occur; it describes the forces (or energy transmission) acting on the body that cause injury. Index of suspicion is your concern for potentially

During the rapid head-to-toe assessment of a patient with multiple injuries, you expose the chest and find an open wound with blood bubbling from it. You should:
� A:apply high-flow supplemental oxygen.
� B:prevent air from entering the wound.
� C:place a

You selected B; This is correct!
Reason:
A sucking chest wound (open pneumothorax) is a life-threatening condition that must be corrected immediately upon discovery. You must take immediate action to prevent air from entering the wound or the patient's co

Which of the following types of stroke would MOST likely present with a sudden, severe headache?
� A:Hemorrhagic
� B:Ischemic
� C:Thrombotic
� D:Embolic

You selected A; This is correct!
Reason:
A stroke occurs when blood flow to a portion of the brain is interrupted. There are two types of stroke: ischemic and hemorrhagic, both of which are acute events. Hemorrhagic stroke is caused by a ruptured artery i

A 33-year-old factory worker was pinned between two pieces of machinery. When you arrive at the scene, you find him lying supine on the ground complaining of severe pain to his pelvis. He is restless, diaphoretic, and tachycardic. After performing a rapid

You selected C; This is correct!
Reason:
Based on the mechanism of injury and the presence of signs of shock (eg, restlessness, tachycardia, diaphoresis), you should suspect that the patient has a fractured pelvis and is bleeding internally. Therefore, af

During the inhalation phase of breathing:
� A:pressure within the thorax decreases and air is drawn into the lungs.
� B:the diaphragm and intercostal muscles contract and ascend.
� C:air passively enters the lungs as pressure within the thorax increases.

You selected A; This is correct!
Reason:
Inhalation is the active, muscular part of breathing. During inhalation, the diaphragm and intercostal muscles contract. When the diaphragm contracts, it moves down (descends) slightly and enlarges the thoracic cag

An elderly woman, who was removed from her burning house by firefighters, has full-thickness burns to approximately 50% of her body. Appropriate treatment for this patient should include:
� A:peeling burned clothing from the skin and removing all rings, n

The correct answer is D;
Reason:
After moving the patient to safety, stopping the burning process, and supporting the ABCs, full-thickness burns should be cared for by applying dry, sterile dressings or burn pads and preventing hypothermia. Cooling full-t

After administering nitroglycerin to a patient with chest discomfort, it is MOST important for you to:
� A:place the patient supine and elevate his or her legs.
� B:ask the patient if the discomfort has improved.
� C:reassess his or her blood pressure wit

You selected C; This is correct!
Reason:
Nitroglycerin (NTG) relaxes the muscle of blood vessel walls, dilates the coronary arteries, increases blood flow and the supply of oxygen to the heart muscle (myocardium), and decreases the workload of the heart.

A 22-year-old female fell on her knee and is in severe pain. Her knee is flexed and severely deformed. Her leg is cold to the touch and you are unable to palpate a distal pulse. You should:
� A:carefully straighten her leg until you restore a distal pulse

The correct answer is C;
Reason:
A dislocated knee occurs when the proximal end of the tibia completely displaces from its juncture with the distal femur. In some cases, the popliteal artery behind the knee may be compressed, resulting in compromised dist

Which of the following is a common side effect of nitroglycerin?
� A:Nausea
� B:Headache
� C:Hypertension
� D:Anxiety

You selected B; This is correct!
Reason:
Because nitroglycerin (NTG) causes vasodilation, including the vessels within the brain, cerebral blood flow increases following the administration of NTG. This often causes a pounding headache for the patient. As

While assessing a patient with chest pain, you note that his pulse is irregular. This indicates:
� A:acute myocardial infarction or angina pectoris.
� B:high blood pressure that is increasing cardiac workload.
� C:a dysfunction in the left side of the pat

You selected D; This is correct!
Reason:
An irregular pulse indicates abnormalities in the electrical conduction system of the heart. The electrical conduction system, beginning with the sinoatrial node as the primary pacemaker, is responsible for initiat

Which of the following statements regarding the automated external defibrillator (AED) is correct?
� A:AEDs can safely be used in infants and children less than 8 years of age
� B:AEDs will analyze the patient's rhythm while CPR is in progress
� C:The AED

You selected A; This is correct!
Reason:
According to the 2010 guidelines for CPR and Emergency Cardiac Care (ECC), the AED can safely be used in infants and children less than 8 years of age. Although a manual defibrillator is preferred in infants, an AE

Which of the following is an abnormal finding when using the Cincinnati stroke scale to assess a patient who presents with signs of a stroke?
� A:One arm drifts down compared with the other side.
� B:Both arms drift slowly and equally down to the patient'

The correct answer is A;
Reason:
The Cincinnati Stroke Scale is used to assess patients suspected of experiencing a stroke. It consists of three tests: speech, facial droop, and arm drift. Abnormality in any one of these areas indicates a high probability

Shortly after assisting a 60-year-old woman with her second nitroglycerin treatment, she tells you that she is lightheaded and feels like she is going to faint. Her symptoms are MOST likely due to:
� A:low blood pressure.
� B:an irregular heartbeat.
� C:a

You selected A; This is correct!
Reason:
Nitroglycerin (NTG) is a vasodilator; as such, it may cause a drop in blood pressure (hypotension) in some patients. Signs and symptoms of hypotension include dizziness, lightheadedness, and fainting (syncope), amo

Shock following major trauma is MOST often the result of:
� A:head injury.
� B:long bone fractures.
� C:spinal injury.
� D:hemorrhage.

You selected D; This is correct!
Reason:
Shock following major trauma is usually caused by hemorrhage (bleeding), which can be external and obvious (gross), internal and hidden (occult), or both. Trauma to the chest and/or abdomen and multiple long bone f

An unresponsive patient's respirations are 26 breaths/min and shallow. The MOST appropriate treatment includes:
� A:a simple face mask set at 10 to 12 L/min.
� B:a nonrebreathing mask set at 15 L/min.
� C:assisted ventilations with 100% oxygen.
� D:a nasa

You selected C; This is correct!
Reason:
Shallow respirations (reduced tidal volume) at a rate of 26 breaths/min will not provide adequate minute volume. Therefore, you should assist the patient's ventilations with a bag-mask device and high-flow oxygen.

Prior to administering nitroglycerin to a patient with chest pain, you should:
� A:elevate the patient's lower extremities.
� B:obtain vital signs to detect hypotension.
� C:auscultate the patient's breath sounds.
� D:inquire about an allergy to salicylat

You selected B; This is correct!
Reason:
Prior to assisting a patient with his or her prescribed nitroglycerin, there are two things that you must do: take the patient's vital signs and obtain authorization from medical control. Nitroglycerin is contraind

The method by which you administer supplemental oxygen to a hypoxic patient depends MOSTLY on the:
� A:suspected underlying cause of the hypoxia.
� B:presence or absence of cyanosis.
� C:patient's level of consciousness and heart rate.
� D:severity of hyp

You selected D; This is correct!
Reason:
All hypoxic patients, whatever the cause of their condition, should be treated with high-flow oxygen. The method of oxygen delivery depends on the severity of the hypoxia and the adequacy of the patient's breathing

A 19-year-old male was assaulted and has trauma to multiple body systems. After performing your primary assessment and treating any immediate life-threatening injuries, you should:
� A:obtain a full set of baseline vital signs.
� B:transport at once and i

You selected D; The correct answer is C;
Reason:
After treating all life-threatening conditions found in the primary assessment, you should perform a rapid head-to-toe assessment (rapid body scan) to look for and treat other life threats. In many cases, p

A middle-aged male was found unresponsive by his wife. When you arrive at the scene, you assess the patient and determine that he is apneic and pulseless. You should:
� A:perform CPR with a compression-to-ventilation ratio of 15:2, apply the AED, and requ

You selected D; This is correct!
Reason:
As soon as you determine that a patient is unresponsive, pulseless, and apneic, you should begin CPR starting with chest compressions. The appropriate compression-to-ventilation ratio for adult CPR (one- or two-res

After defibrillating a man in cardiac arrest, you resume CPR. As you are about to reanalyze his cardiac rhythm 2 minutes later, your partner tells you she can definitely feel a strong carotid pulse. You should:
� A:remove the AED pads from the patient's c

You selected D; This is correct!
Reason:
If return of spontaneous circulation (ROSC) occurs, your first action should be to reassess the patient's airway status and breathing effort. If the patient is still apneic or is breathing inadequately, continue ve

If an adult patient presents with a respiratory rate of 26 breaths/min, your initial action should be to:
� A:evaluate his mental status and the depth of his respirations.
� B:begin assisting his ventilations with a bag-mask device.
� C:apply oxygen via a

You selected A; This is correct!
Reason:
The normal respiratory rate for an adult at rest is 12 to 20 breaths/min. If a patient presents with a respiratory rate outside of the normal range, you should immediately assess him or her for other signs of inade

Freshly oxygenated blood returns to the heart via the:
� A:aorta.
� B:vena cavae.
� C:pulmonary artery.
� D:pulmonary vein.

You selected B; The correct answer is D;
Reason:
The pulmonary vein is the only vein that carries oxygen-rich blood. It carries blood from the lungs back to the left atrium. All other veins in the human body, including the vena cavae, carry deoxygenated b

You are assessing a middle-aged male who is experiencing respiratory distress. The patient has a history of emphysema and hypertension. He appears fatigued; has weak retractions; and labored, shallow breathing. Your MOST immediate action should be to:
� A

You selected C; This is correct!
Reason:
Your patient is NOT breathing adequately. He is fatigued; has weak retractions; and labored, shallow breathing. If you do not treat him immediately, he may stop breathing altogether. You should begin assisting his

After stopping the burning process, emergency care for a 68-year-old male with partial- and full-thickness burns to his chest and upper extremities includes all of the following, EXCEPT:
� A:preparing to assist the patient's ventilations.
� B:covering the

You selected C; This is correct!
Reason:
Unless the patient is on fire, do not apply water to a full-thickness (third-degree) burn, especially if the patient is already prone to hypothermia and infection (ie, older adults, small children). Cover the burns

A 30-year-old woman has an open deformity to her left leg and is in severe pain. She is conscious and alert, has a patent airway, and is breathing adequately. Your primary concern should be:
� A:administering high-flow oxygen.
� B:controlling any external

The correct answer is B;
Reason:
Initial care for any open injury involves controlling external bleeding. Further care involves manually stabilizing the injury site; applying a sterile dressing to keep gross contaminants from entering the wound; assessing

A patient's skin will MOST likely become cyanotic if he or she has:
� A:a decrease in the amount of carbon dioxide.
� B:a decrease in the amount of arterial oxygen.
� C:an increase in the amount of arterial oxygen.
� D:an overall increase in circulating r

You selected B; This is correct!
Reason:
Cyanosis, a blue or purple tint to the skin, reflects an inadequate amount of oxygen in the arterial blood. More specifically, cyanosis indicates that a significant amount of hemoglobin has separated from the red b

Which of the following occurs during positive-pressure ventilation?
� A:Intrathoracic pressure increases
� B:The esophagus remains closed
� C:Oxygen is pulled into the lungs
� D:Blood is drawn back to the heart

You selected A; This is correct!
Reason:
Negative-pressure ventilation, the act of normal breathing, occurs when the diaphragm and intercostal muscles contract. The actions of these muscles create a vacuum (negative pressure), which pulls oxygen-rich air

A gang member was cut on the left side of the neck during a fight and is bleeding heavily from the wound. His airway is patent and his breathing is adequate. You should immediately:
� A:cover the wound with an occlusive dressing and apply direct pressure.

You selected A; This is correct!
Reason:
Neck lacerations are extremely dangerous and can result in severe bleeding and shock, air embolism, or both. If a jugular vein is lacerated, air can be sucked into the wound, enter the circulatory system, and cause

An unresponsive man has shallow, gurgling respirations at a rate of 8 breaths/min. Initial treatment should include:
� A:oxygen via nonrebreathing mask.
� B:positive-pressure ventilations.
� C:suctioning of the oropharynx.
� D:oropharyngeal airway inserti

You selected D; The correct answer is C;
Reason:
Before breathing can be assessed, let alone managed, the airway must be cleared of any and all secretions. When you hear gurgling respirations, you should provide immediate suctioning of the oropharynx for

You are assessing a 70-year-old male who complains of pain in both of his legs. He is conscious and alert, has a blood pressure of 160/90 mm Hg, a pulse rate of 110 beats/min, and respirations of 14 breaths/min and unlabored. Further assessment reveals ed

You selected D; This is correct!
Reason:
If the right side of the heart is damaged, fluid collects in the body (edema), often showing in the feet and legs. The collection of fluid in the part of the body that is closest to the ground is called dependent e

Damaged small blood vessels beneath the skin following blunt trauma causes:
� A:mottling.
� B:ecchymosis.
� C:hematoma.
� D:cyanosis.

You selected B; This is correct!
Reason:
When small blood vessels beneath the skin are damaged, blood seeps into the soft tissues. This manifests as a bruise, also referred to as ecchymosis. A hematoma develops when larger blood vessels are ruptured and t

You are ventilating an apneic 50-year-old woman with a bag-mask device. After squeezing the bag and noting visible chest rise, you should:
� A:allow the patient to completely exhale.
� B:squeeze the bag again in 3 seconds.
� C:reopen the airway and ventil

You selected A; This is correct!
Reason:
When ventilating an apneic patient, it is important to allow for complete exhalation. To do this, deliver each breath over 1 second, just enough to make the chest visibly rise, and then deliver the next breath 5 to

When performing two-rescuer CPR on an adult patient whose airway has not been secured with an advanced device, you should:
� A:continue ventilations as the AED analyzes the patient's cardiac rhythm.
� B:deliver ventilations at a rate of 8 to 10 breaths/mi

You selected C; This is correct!
Reason:
When performing two-rescuer adult CPR, you should perform cycles of CPR, with a compression to ventilation ratio of 30:2. If the airway is not secured with an advanced device (eg, ET tube, multilumen airway, suprag

You should be MOST suspicious that a patient with chest pressure has an underlying cardiac problem if his or her pulse is:
� A:slow.
� B:irregular.
� C:rapid.
� D:bounding.

You selected B; This is correct!
Reason:
Of the choices listed, an irregular pulse should make you the most suspicious that a patient with chest pain, pressure, or discomfort has an underlying cardiac problem. An irregular pulse indicates a cardiac dysrhy

A 50-year-old man, who fell approximately 20 feet and landed on a hard surface, is semiconscious. You should:
� A:gently tilt the patient's head back to assess for breathing.
� B:stabilize his head while performing the jaw-thrust maneuver.
� C:check for a

You selected B; This is correct!
Reason:
Because of the significant mechanism of injury (fall of greater than 15 feet), spinal injury should be assumed. The first step in managing this patient is to manually stabilize his head in a neutral position and op

A 44-year-old male experienced burns to his anterior trunk and both arms. He is conscious and alert, but is in extreme pain. Assessment of the burns reveals reddening and blisters. This patient has ________________ burns that cover _____ of his total body

The correct answer is D;
Reason:
Partial-thickness (second-degree) burns damage the epidermis and part of the dermis, and are characterized by blistering and severe pain. Areas of superficial (first-degree) burns, which cause reddening of the skin, common

When ventilating an apneic patient with a pocket mask device, each breath should be delivered over:
� A:3 seconds.
� B:4 seconds.
� C:1 second.
� D:2 seconds.

You selected C; This is correct!
Reason:
When ventilating any apneic patient, each breath should be delivered over a period of 1 second�just enough to produce visible chest rise. Excessive ventilation duration and/or volume increases the likelihood of gas

Internal or external bleeding would be especially severe in a patient:
� A:with hemophilia.
� B:with heart disease.
� C:who is hypotensive.
� D:who takes aspirin.

You selected A; This is correct!
Reason:
Hemophilia is a condition in which the patient lacks one or more of the blood's clotting factors. There are several forms of hemophilia, most of which are hereditary and some of which are severe. Sometimes bleeding

Which of the following interventions would the EMT be the LEAST likely to perform while attempting to resuscitate a cardiac arrest patient?
� A:Insertion of a supraglottic airway device.
� B:Assisting a paramedic with intubation
� C:Rhythm analysis with t

The correct answer is A;
Reason:
The insertion of advanced airway devices (eg, endotracheal [ET] tube, multilumen airway, supraglottic airway) is generally outside the EMT's scope of practice. However, he or she may be asked to assist a paramedic in the p

A nonrebreathing mask is MOST appropriate to use on patients who:
� A:are semiconscious and breathing shallowly.
� B:are cyanotic and have a low oxygen saturation.
� C:are breathing less than 12 times per minute.
� D:have an adequate rate and depth of bre

You selected D; This is correct!
Reason:
With the oxygen flow rate set at 15 L/min, the nonrebreathing mask can deliver an oxygen concentration of 90% or greater. Unlike the bag-mask or pocket mask devices, which deliver oxygen via positive pressure, the

A man was struck in the side of the head with a steel pipe. Blood-tinged fluid is draining from the ear and bruising appears behind the ear. The MOST appropriate treatment for this patient includes:
� A:controlling the drainage from the ear and immobilizi

You selected C; The correct answer is D;
Reason:
Patients with significant head injury should be treated by applying high-flow oxygen, assisting ventilations as needed, immobilzing the entire spine, and transporting promptly. Closely monitor the patient f

A patient reports pain in the upper midabdominal area. This region of the abdomen is called the:
� A:retroperitoneum.
� B:peritoneum.
� C:mediastinum.
� D:epigastrium.

The correct answer is D;
Reason:
The mid-upper region of the abdomen is referred to as the epigastrium because of its location over the stomach (epi = upon, gastric = stomach). This is a common site of pain or discomfort in patients experiencing a cardiac

Which of the following patients has signs of inadequate breathing?
� A:A 41-year-old woman with shallow respirations of 14 breaths/min
� B:A 30-year-old man with respirations of 12 breaths/min with adequate depth
� C:A 50-year-old woman with respirations

You selected A; This is correct!
Reason:
Although the 41-year-old woman has a respiratory rate that falls within the normal range for an adult, the depth of her breathing is shallow (reduced tidal volume). Signs of inadequate breathing in the adult includ

A patient with a spinal injury may still be able to use his or her diaphragm to breathe, but would lose control of the intercostal muscles, if the spinal cord is injured:
� A:above the C5 level.
� B:below the C5 level.
� C:above the C3 level.
� D:between

You selected B; This is correct!
Reason:
The nerves that supply the diaphragm (the phrenic nerves) exit the spinal cord at C3, C4, and C5. A patient whose spinal cord is injured below the C5 level will lose the ability to move his or her intercostal muscl

You have analyzed a cardiac arrest patient's rhythm three times with the AED, separated by 2-minute cycles of CPR, and have received no shock messages each time. You should:
� A:consider terminating resuscitation.
� B:continue CPR and transport at once.

You selected B; This is correct!
Reason:
Although protocols vary from system to system, it is generally agreed that if you receive three consecutive no shock messages, separated by 2-minute cycles of CPR, you should continue CPR and transport at once; it

A 33-year-old male struck a parked car with his motorcycle and was ejected from the motorcycle. He was not wearing a helmet. He is unresponsive, has a depressed area to his forehead, bilaterally deformed femurs, and widespread abrasions with capillary ble

You selected B; This is correct!
Reason:
The patient's abrasions (road rash) and capillary bleeding are the least of his problems. Capillary bleeding, blood that oozes from the capillary beds, is the least severe type of external bleeding and will not kil

Ischemic heart disease is a condition in which:
� A:an acute event leads to a significant decrease in the pumping force of the heart.
� B:a portion of the heart muscle dies because of a prolonged lack of oxygen.
� C:the coronary arteries dilate, thus prev

You selected B; The correct answer is D;
Reason:
Chest pain or discomfort that is related to the heart usually stems from a condition called ischemia (insufficient oxygen). Because of a partial or complete blockage of blood flow through one or more corona

Which of the following assessment parameters is the MOST reliable when determining if a patient with a head injury is improving or deteriorating?
� A:pupillary reaction.
� B:level of consciousness.
� C:systolic blood pressure.
� D:rate and depth of breath

You selected B; This is correct!
Reason:
All of the options in this question are important parameters to monitor in a patient with a head injury. However, the single most reliable parameter is the patient's level of consciousness (LOC); a person's LOC ind

When attaching an oxygen regulator to a D cylinder and preparing it for use, you should recall that:
� A:the cylinder must remain in a standing position at all times or it will not deliver any oxygen.
� B:a pressure-compensated flowmeter should be used wh

You selected C; This is correct!
Reason:
Oxygen does not burn or explode; however, it does support combustion. A small spark, even a lit cigarette, can become a flame in an oxygen-rich atmosphere. Therefore, you must ensure that the environment in which y

Which of the following is MOST indicative of decompensated shock in a trauma patient with internal bleeding?
� A:Restlessness
� B:Hypotension
� C:Clammy skin
� D:Tachycardia

You selected B; This is correct!
Reason:
In compensated shock, the body is able to maintain blood pressure, usually above 90 to 100 mm Hg, through the physiologic responses of tachycardia and shunting of blood from the skin to the vital organs of the body

If the level of carbon dioxide in the arterial blood increases:
� A:a reduction in tidal volume will occur.
� B:the respiratory rate and depth decrease.
� C:the respiratory rate slows significantly.
� D:the respiratory rate and depth increase.

You selected D; This is correct!
Reason:
Special receptors, called chemoreceptors, sense the levels of oxygen and carbon dioxide in the arterial blood. The central chemoreceptors are located in the brain; the peripheral chemoreceptors are located in the a

Which of the following injuries or conditions should be managed FIRST?
� A:Fluid drainage from both ears
� B:A large open abdominal wound
� C:Bilateral fractures of the femurs
� D:Bleeding within the oral cavity

You selected D; This is correct!
Reason:
Any injury or condition that jeopardizes the airway has priority over all else. If blood or other secretions within the mouth are not suctioned immediately, aspiration may occur; this significantly increases mortal

Which of the following patients is the BEST candidate for an oropharyngeal airway?
� A:An unresponsive trauma patient with blood draining from the nose
� B:Any patient that you suspect of being acutely hypoxemic
� C:An unresponsive patient with uncontroll

You selected A; This is correct!
Reason:
The oropharyngeal airway is used to keep the tongue off of the posterior pharynx and is indicated for unresponsive patients without a gag reflex. If an unresponsive patient has severe, uncontrolled oropharyngeal bl

A young woman who overdosed on heroin is unresponsive with slow, shallow breathing. As you attempt to insert an oropharyngeal airway, she begins to gag. You should:
� A:place her on her side until she stops gagging and then suction her mouth.
� B:remove t

You selected B; This is correct!
Reason:
Although uncommon, an unresponsive patient may have an active gag reflex. If an unresponsive patient begins to gag as you are attempting to insert an oropharyngeal airway, you must remove the airway immediately and

You are dispatched to the scene of a motorcycle crash. Upon arrival, you find the patient lying facedown approximately 25 feet from his bike. He is not wearing a helmet and is moaning. You should:
� A:log roll him to a supine position.
� B:apply a cervica

You selected B; The correct answer is C;
Reason:
The mechanism of injury for this patient was significant. In his present position (prone), you cannot effectively assess his airway. Therefore, your first action should be to manually stabilize his head. Th

Following blunt trauma to the chest, a 33-year-old male has shallow, painful breathing. On assessment, you note that an area to the left side of his chest collapses during inhalation and bulges during exhalation. These are signs of a/an:
� A:flail chest.

You selected A; This is correct!
Reason:
If two or more ribs are fractured in two or more places or if the sternum is fractured along with several ribs, a segment of chest wall may be detached from the rest of the thoracic cage. This injury is called a fl

Oxygen that is administered through a nasal cannula would be of LEAST benefit to a patient who:
� A:is breathing greater than 12 times per minute.
� B:breathes through his or her mouth.
� C:has COPD and an oxygen saturation of 94%.
� D:is in need of long-

You selected B; This is correct!
Reason:
A patient who breathes through the mouth or has a nasal obstruction will get little or no benefit from a nasal cannula. Many patients with COPD (eg, emphysema, chronic bronchitis) require long-term, low-flow oxygen

Which of the following signs or symptoms is indicative of cerebral hypoxia?
� A:Decreased level of consciousness
� B:Heart rate greater than 120 beats/min
� C:Diffuse wheezing on exhalation
� D:Chief complaint of dyspnea

You selected A; This is correct!
Reason:
Dyspnea, a feeling of shortness of breath, is a symptom of a condition that can cause cerebral hypoxia (eg, CHF, COPD); however, dyspnea itself does not indicate cerebral hypoxia. Wheezing, a whistling sound that i

The presence of subcutaneous emphysema following blunt trauma to the anterior neck should make you MOST suspicious for a:
� A:pneumothorax.
� B:carotid artery injury.
� C:fractured larynx.
� D:ruptured esophagus.

You selected A; The correct answer is C;
Reason:
Crushing or blunt trauma to the anterior neck can injure the trachea or larynx. Once the cartilages of the upper airway and larynx are fractured, they do not spring back to their normal position. Such a fra

Your partner has applied the AED to a cardiac arrest patient and has received a shock advised message. While the AED is charging, you should:
� A:retrieve the airway equipment and prepare to ventilate the patient.
� B:continue chest compressions until you

You selected B; This is correct!
Reason:
It is important to minimize interruptions in CPR, especially chest compressions, when at all possible. All contact with the patient must cease while the AED is analyzing. However, if the AED gives a shock advised m

A young male has a large laceration to his lateral neck, directly over his jugular vein. His airway is patent and his breathing is adequate. Your MOST immediate priority should be to:
� A:obtain vital signs to determine if he is hypotensive.
� B:keep air

You selected B; This is correct!
Reason:
Jugular vein lacerations pose two immediate life threats: entrainment of air into the wound (which may cause a fatal air embolism) and severe external bleeding. The patient's airway is patent and his breathing is a

You are dispatched to a residence for an elderly female who has possibly suffered a stroke. You find her lying supine in her bed. She is semiconscious; has vomited; and has slow, irregular breathing. You should:
� A:perform a head tilt-chin lift and inser

You selected C; This is correct!
Reason:
This patient's airway is in immediate jeopardy! The first step in caring for any semi- or unconscious patient is to manually open the airway (eg, head tilt-chin lift, jaw-thrust) and ensure it is clear of obstructi

After delivering one shock with the AED and performing 2 minutes of CPR on a woman in cardiac arrest, you reanalyze her cardiac rhythm and receive a no shock advised message. This means that:
� A:the first shock restored a rhythm and pulse.
� B:she has el

You selected C; This is correct!
Reason:
If the AED gives a no shock advised message, it has determined that the patient is not in a shockable rhythm (eg, V-Fib, pulseless V-Tach). It does not indicate that the patient has a pulse, nor does it indicate th

You are performing abdominal thrusts on a 19-year-old male with a severe airway obstruction when he becomes unresponsive. After lowering him to the ground and placing him in a supine position, you should:
� A:assess for a carotid pulse for up to 10 second

You selected D; This is correct!
Reason:
A patient with a severe airway obstruction may initially be responsive and then become unresponsive during treatment. In this case, you know that an airway obstruction is the cause of his or her problem. Therefore,

By which of the following mechanisms does nitroglycerin relieve cardiac-related chest pain or discomfort?
� A:Vasodilation and increased myocardial oxygen supply
� B:Vasoconstriction and increased cardiac oxygen demand
� C:Vasoconstriction and increased m

You selected A; This is correct!
Reason:
Nitroglycerin is a smooth muscle relaxant. Smooth muscle is found within the walls of the blood vessels. Nitroglycerin causes vasodilation, including dilation of the coronary arteries, which in turn increases the f

After an initial attempt to ventilate an unresponsive apneic patient fails, you reposition the patient's head and reattempt ventilation without success. You should next:
A: turn the patient onto his side and deliver 5 to 10 back slaps.
B: perform chest co

You selected B; This is correct!
Reason: If you are unable to ventilate an unresponsive, apneic patient after two attempts, you should assume that he or she has a severe (complete) foreign body airway obstruction. Immediately perform 30 chest compressions

A 23-year-old unrestrained female struck the steering wheel with her chest when her passenger car collided with a tree at a high rate of speed. Your assessment reveals that she is conscious, but has signs of shock and an irregular pulse. The MOST appropri

You selected D; This is correct!
Reason:
The cause of this patient's shock may be a myocardial contusion, or bruising of the heart muscle. Blunt trauma to the chest can injure the heart, making it unable to maintain adequate blood pressure. In a myocardia

In patients with heart disease, acute coronary syndrome is MOST often the result of:
� A:coronary artery spasm.
� B:coronary artery rupture.
� C:atherosclerosis.
� D:atrial damage.

You selected C; This is correct!
Reason:
In most patients with acute coronary syndrome, or ACS (eg, unstable angina, acute myocardial infarction), atherosclerosis is the underlying problem that causes heart disease. Atherosclerosis is a disorder in which

Tachycardia can be detrimental to a patient who is experiencing a cardiac problem because it causes:
� A:increased cardiac oxygen usage and demand.
� B:increased cardiac filling in between beats.
� C:a profound decrease in oxygen consumption.
� D:an assoc

You selected A; This is correct!
Reason:
Many patients experiencing a cardiac problem are tachycardic (heart rate greater than 100 beats/min); others are bradycardic (heart rate less than 60 beats/min). As the heart beats faster, it consumes, and therefor

The MOST important initial treatment for a patient whose cardiac arrest was witnessed is:
� A:high-quality CPR.
� B:rapid transport.
� C:defibrillation.
� D:cardiac drug therapy.

You selected A; This is correct!
Reason:
Regardless of whether a patient's cardiac arrest is witnessed or unwitnessed, the single most important initial treatment is high-quality CPR. Delays in performing CPR have been clearly linked to poor patient outco

You are assessing an elderly man with respiratory distress. He is coughing up bloody sputum and has an oxygen saturation of 85%. You auscultate his breath sounds and hear coarse crackles in all lung fields. This patient MOST likely has:
� A:congestive hea

The correct answer is A;
Reason:
This patient's signs and symptoms are classic for left-sided congestive heart failure and pulmonary edema. As the left side of the heart weakens, in which case it can no longer effectively pump blood, blood backs up into t

During your rapid assessment of a critically-injured patient, you should assess the chest for:
� A:crepitus and distention.
� B:distention and guarding.
� C:rigidity and guarding.
� D:symmetry and pain.

You selected D; This is correct!
Reason:
When assessing the chest during the assessment, you should check for symmetry (equal rise of the chest), assess for pain upon palpation, and the presence of equal breath sounds bilaterally. Crepitus also should be

Treatment for a patient with congestive heart failure and shortness of breath may include:
� A:prophylactic suctioning of the airway.
� B:supine positioning and elevation of the legs.
� C:hyperventilation with a bag-mask device.
� D:up to three doses of s

You selected D; This is correct!
Reason:
Treatment for patients with congestive heart failure (CHF) includes supplemental oxygen as needed to maintain an oxygen saturation equal to or greater than 94%, continuous positive airway pressure (CPAP), ventilato

A 50-year-old man presents with crushing chest pain of sudden onset. He is diaphoretic, apprehensive, and tachypneic. You should:
� A:obtain baseline vital signs.
� B:ask him if he takes nitroglycerin.
� C:perform a complete physical exam.
� D:apply suppl

You selected D; This is correct!
Reason:
All of the interventions and assessments listed in this question should be performed on a patient who presents with chest pain, pressure, or discomfort. However, supplemental oxygen is indicated for any patient wit

You arrive at a residence where you find a man lying unresponsive in his front yard. There were no witnesses to the event. In assessing this man, you should assume that he:
� A:has a heat-related emergency.
� B:has sustained an injury.
� C:is having a dia

You selected B; This is correct!
Reason:
In the absence of any witnesses, you should assume that any patient who is found unresponsive has an injury until ruled out at the hospital. Apply spinal motion restriction precautions as needed. Do not be so hasty

Priority treatment for a large avulsion includes:
� A:cleaning the wound.
� B:assessing distal circulation.
� C:immobilizing the injured area.
� D:controlling any bleeding.

You selected D; This is correct!
Reason:
Immediate treatment for any soft-tissue injury begins with controlling any external bleeding. Once the bleeding is controlled, distal circulation, motor, and sensory functions should be assessed, the wound dressed

During your initial attempt to ventilate an unresponsive apneic patient, you meet resistance and do not see the patient's chest rise. You should:
� A:reposition the head and reattempt to ventilate.
� B:begin CPR, starting with chest compressions.
� C:suct

You selected A; This is correct!
Reason:
If your initial attempt to ventilate a patient is met with resistance and/or does not make the chest visibly rise, you should reposition the patient's head and reattempt to ventilate. In many cases, this simple act

Which of the following patients obviously needs positive-pressure ventilation assistance?
� A:Responsive to pain only; respiratory rate of 8 breaths/min and shallow
� B:Combative; respiratory rate of 24 breaths/min and deep
� C:Restless; respiratory rate

You selected A; This is correct!
Reason:
Any patient with a decreased level of consciousness should be assessed for inadequate breathing (eg, fast or slow respiratory rate, reduced tidal volume [shallow breathing]). Of the patients listed, the patient who

A 56-year-old man has labored, shallow breathing at a rate of 28 breaths/min. He is responsive to pain only. You should:
� A:insert a nasopharyngeal airway and begin assisting his ventilations.
� B:place him on his side and administer oxygen via nonrebrea

You selected A; This is correct!
Reason:
This patient in this scenario is not breathing adequately. He is responsive to pain only, and his respirations are rapid, labored, and shallow. You should insert a nasopharyngeal airway, which is usually well-toler

When an error occurs while using the AED, it is MOST often the result of:
� A:malfunction of the microchip inside the AED.
� B:excess patient movement during the analyze phase.
� C:misinterpretation of the patient's cardiac rhythm.
� D:battery failure sec

You selected D; This is correct!
Reason:
AEDs have a high specificity for recognizing shockable rhythm (eg, V-Fib, pulseless V-Tach); this means that they are highly reliable. It would be extremely rare for the AED to recommend a shock when one is not ind

Patients with a hypoxic drive:
� A:are accustomed to low levels of carbon dioxide in the blood.
� B:may hypoventilate if given low concentrations of oxygen.
� C:rarely become cyanotic because of high blood oxygen levels.
� D:are stimulated to breathe by l

You selected D; This is correct!
Reason:
Patients with chronic respiratory diseases (eg, emphysema) maintain decreased levels of oxygen and increased levels of carbon dioxide in the blood. The sensors in the brain become accustomed to this. Unlike a healt

In which of the following patients is nitroglycerin contraindicated?
� A:66-year-old female with chest pressure of 6 hours' duration, lightheadedness, and a blood pressure of 110/58 mm Hg
� B:41-year-old male with crushing substernal chest pressure, a blo

You selected C; This is correct!
Reason:
Nitroglycerin is contraindicated in patients who do not have a prescription for nitroglycerin, in those with a systolic BP less than 100 mm Hg, and in patients who have taken medications for erectile dysfunction (E

A patient has severe facial injuries, inadequate breathing, and copious secretions coming from the mouth. How should this situation be managed?
� A:Alternate suctioning for 15 seconds and ventilations for 2 minutes.
� B:Insert an oropharyngeal airway and

You selected A; This is correct!
Reason:
Both inadequate breathing and secretions in the mouth (ie, blood, vomitus, etc) must be addressed simultaneously. This is best accomplished by suctioning in 15-second increments, then providing assisted ventilation

The AED analyzes your pulseless and apneic patient's cardiac rhythm and advises that a shock is NOT indicated. You should:
� A:resume CPR, starting with chest compressions.
� B:open the patient's airway and check for breathing.
� C:reanalyze the cardiac r

You selected A; This is correct!
Reason:
If the AED advises "no shock," you should immediately resume CPR, starting with chest compressions. Unless the patient starts to move or has other signs of life (ie, coughing), stopping CPR to assess for a pulse sh

Tidal volume is defined as the:
� A:volume of air moved in and out of the lungs each minute.
� B:total volume of air that the lungs are capable of holding.
� C:volume of air inhaled or exhaled per breath.
� D:volume of air that remains in the upper airway

You selected C; This is correct!
Reason:
Tidal volume (VT) is the amount of air that is inhaled or exhaled per breath; it is normally 500 mL in a healthy adult male. Tidal volume is assessed by noting the depth of a patient's breathing. Shallow breathing,

When treating a patient with chest pain, pressure, or discomfort, you should first:
� A:request an ALS ambulance response to the scene.
� B:administer supplemental oxygen.
� C:assess the blood pressure and give nitroglycerin.
� D:place the patient in a po

The correct answer is D;
Reason:
An important aspect of treating a patient with chest pain, pressure, or discomfort is to ensure that the patient is in a comfortable position. Most of the time, the patient will already be in this position upon your arriva

When applying a vest-style spinal immobilization device to a patient with traumatic neck pain, you should:
� A:gently flex the head forward as you position the device.
� B:secure the torso section prior to immobilizing the head.
� C:immobilize the head pr

You selected B; This is correct!
Reason:
When you apply a vest-style immobilization device such as a KED, you must immobilize the patient's head after the torso is adequately secured. If you immobilize the head first, the cervical spine may be unnecessari

Which of the following patients is the BEST candidate for the administration of nitroglycerin?
� A:A man with chest pain, expired nitroglycerin spray, and a blood pressure of 110/80 mm Hg
� B:A woman who has taken three doses of prescribed nitroglycerin w

You selected C; This is correct!
Reason:
Nitroglycerin should be administered to patients who have the prescribed, unexpired drug with them and a systolic blood pressure of greater than 100 mm Hg. No more than three (3) nitroglycerin tablets or sprays sho

Basic shock treatment includes:
� A:applying oxygen, elevating the lower extremities per protocol, and providing warmth.
� B:elevating the lower extremities, applying and inflating the PASG, and applying oxygen.
� C:applying and inflating the PASG, applyi

You selected A; This is correct!
Reason:
Basic shock treatment, which should be initiated as soon as possible, includes applying high-flow oxygen, elevating the lower extremities 6 to 12 inches (if dictated by your local protocols), and providing warmth.

When assessing and treating a patient with a gunshot wound, you should routinely:
� A:look for the presence of an exit wound.
� B:determine why the patient was shot.
� C:apply ice directly to the wound.
� D:evaluate the pulses proximal to the wound.

You selected A; This is correct!
Reason:
When assessing a patient who sustained a gunshot wound, you should routinely look for an exit wound, which may be difficult to find. Exit wounds can be a source of continued bleeding, both externally and internally

Which of the following would MOST likely occur if an adult patient is breathing at a rate of 45 breaths/min with shallow depth?
� A:The lungs would become hyperinflated, potentially causing a pneumothorax.
� B:The volume of air that reaches the alveoli wo

You selected C; This is correct!
Reason:
Alveolar minute volume, the amount of air that reaches the alveoli per minute and participates in pulmonary respiration, is affected by tidal volume, respiratory rate, or both. If the respiratory rate decreases, ti

A 56-year-old man is found to be pulseless and apneic. His wife states that he collapsed about 5 minutes ago. As your partner gets the AED from the ambulance, you should:
� A:ask the wife if the patient has a living will.
� B:begin CPR, starting with ches

You selected C; The correct answer is B;
Reason:
When you arrive on scene and determine that a patient is in cardiac arrest, you should immediately begin CPR, starting with chest compressions. Perform 30 chest compressions and then open the airway and del

A 40-year-old man was hit in the nose during a fight. He has bruising under his left eye and a nosebleed. After taking standard precautions, you should:
� A:place a chemical icepack over his nose.
� B:apply direct pressure by pinching his nostrils togethe

You selected C; This is correct!
Reason:
During a nosebleed (epistaxis), much of the blood may pass down the throat into the stomach as the patient swallows; this is especially true if the patient is lying supine. Blood is a gastric irritant; a person who

Sudden cardiac arrest in the adult population is MOST often the result of:
� A:myocardial infarction.
� B:a cardiac dysrhythmia.
� C:an acute stroke.
� D:respiratory failure.

You selected A; The correct answer is B;
Reason:
Most cases of sudden cardiac arrest (SCA) in the adult (70% to 75%) are the result of a cardiac dysrhythmia, most commonly ventricular fibrillation (V-Fib). This fact underscores the importance of early def

An inaccurate pulse oximetry reading may be caused by:
� A:excessive red blood cell production.
� B:severe peripheral vasoconstriction.
� C:a heart rate greater than 100 beats/min.
� D:heat illnesses, such as heat stroke.

You selected A; The correct answer is B;
Reason:
A pulse oximeter measures the percentage of hemoglobin that is saturated with oxygen. Under normal conditions, a patient's oxygen saturation (SpO2) ranges between 95% and 100% while breathing room air. Alth

Sudden cardiac arrest in the adult population MOST often is the result of:
� A:accidental electrocution.
� B:myocardial infarction.
� C:respiratory failure.
� D:a cardiac arrhythmia.

You selected D; This is correct!
Reason:
The most common cause of sudden cardiac arrest (SCA) in the adult population is a cardiac arrhythmia�usually ventricular fibrillation�in up to 70% of cases. This fact underscores the importance of early defibrillat

A patient with a mild foreign body airway obstruction:
� A:has progressive difficulty breathing.
� B:is typically not cyanotic.
� C:presents with a weak cough.
� D:has a low oxygen saturation.

The correct answer is B;
Reason:
Patients with a mild (partial) airway obstruction are able to move adequate amounts of air, but will have varying degrees of respiratory distress. The patient can cough forcefully, although you may hear wheezing in between

Rapid extrication of a patient from an automobile should be performed by:
� A:applying a vest-style extrication device and sliding the patient out of the car onto a long spine board for full immobilization.
� B:maintaining support of the head, grasping th

The correct answer is C;
Reason:
Unless there is an immediate threat to your or the patient's life, you should perform a rapid extrication by applying a cervical collar, sliding a long spine board under the patient's buttocks, turning and placing the pati

Which of the following statements regarding sudden cardiac arrest and ventricular fibrillation is correct?
� A:Patients with ventricular fibrillation are typically unconscious, apneic, and have a weak and irregular pulse.
� B:For each minute that defibril

You selected B; This is correct!
Reason:
Ventricular fibrillation (V-Fib), a chaotic quivering of the heart muscle, is the most common dysrhythmia that results in sudden cardiac arrest (SCA). In V-Fib, the heart is not beating effectively and is not pumpi

The tidal volume of an unresponsive patient is rapidly assessed by:
� A:auscultating his or her lung sounds.
� B:counting the patient's respiratory rate.
� C:observing for chest rise during inhalation.
� D:evaluating for the presence of cyanosis.

You selected C; This is correct!
Reason:
Tidal volume, a measure of the depth of breathing, is the amount of air in milliliters (mL) that is moved into or out of the lungs during a single breath. The average tidal volume for an adult male is approximately

In which of the following situations should the jaw-thrust maneuver be used?
� A:When the mechanism of injury is unclear
� B:In any patient who is in cardiac arrest
� C:In a patient who is in need of frequent suctioning
� D:In a patient with apnea with no

You selected A; This is correct!
Reason:
The jaw-thrust maneuver should be used to open the airway any time the mechanism of injury suggests trauma or when the mechanism of injury is unclear (ie, in a patient who became unresponsive without witnesses). Wh

You are performing a secondary assessment on a severely injured patient while en route to a trauma center. During the assessment, you note that the patient's respiratory rate has increased. You should:
� A:repeat the primary assessment and treat as needed

You selected A; This is correct!
Reason:
Any time a patient's condition deteriorates, such as your patient whose respirations have increased, you should immediately repeat the primary assessment and adjust your treatment accordingly. For example, a patien

Sonorous respirations are MOST rapidly corrected by:
� A:suctioning the oropharynx.
� B:inserting an oropharyngeal airway.
� C:correctly positioning the head.
� D:initiating assisted ventilations.

You selected C; This is correct!
Reason:
Sonorous (snoring) respirations, which most commonly result from partial airway obstruction by the tongue, are most rapidly corrected by simply positioning the head. This involves using either the head tilt-chin li

A football player complains of severe neck pain and tingling in his arms and legs after being tackled. He is conscious and alert, has a patent airway, and is breathing adequately. He is in a supine position and is still wearing his helmet, which is tight-

The correct answer is A;
Reason:
A helmet that fits well prevents the patient's head from moving and should be left on, provided (1) there are no impending airway or breathing problems, (2) it does not interfere with your assessment and treatment of airwa

An unresponsive patient with multi-systems trauma has slow, shallow breathing; weak radial pulses; and severe bleeding from a lower extremity wound. You should direct your partner to:
� A:assist the patient's ventilations while you control the bleeding.

You selected A; This is correct!
Reason:
The goal of the primary assessment is to rapidly identify and correct all life-threatening injuries or conditions. In the case of this patient, as your partner maintains in-line cervical spine control, he or she sh

You assess an unresponsive 65-year-old man and find that he is apneic and pulseless. The patient's wife tells you that he has an automatic implanted cardioverter/defibrillator. After initiating CPR, you should:
� A:apply the AED as soon as possible and an

You selected A; This is correct!
Reason:
Some patients who are at high risk for sudden cardiac arrest due to ventricular fibrillation (V-Fib) have a small automatic implanted cardioverter/defibrillator (AICD). The AICD attaches directly to the heart and c

General care for an amputated body part includes:
� A:thoroughly cleaning the amputated part and wrapping it in a sterile dressing.
� B:immersing the amputated part in ice cold water to prevent further damage.
� C:wrapping the amputated part in a moist, s

You selected D; This is correct!
Reason:
General care for an amputated body part includes wrapping the part in a moist, sterile dressing and keeping it cool. Placing the wrapped part in a plastic bag and putting it on ice can accomplish this. The amputate

Displaced fractures of the proximal femur are characterized by:
� A:lengthening and internal rotation of the leg.
� B:shortening and external rotation of the leg.
� C:hip joint extension and external leg rotation.
� D:a flexed hip joint and inward thigh r

You selected C; The correct answer is B;
Reason:
Fractures of the proximal (upper) part of the femur are especially common in older people, particularly those with osteoporosis, but may also occur as a result of high-energy trauma in younger patients. Alt

The lower airway begins at the:
� A:trachea.
� B:larynx.
� C:epiglottis.
� D:cricoid cartilage.

You selected B; This is correct!
Reason:
Anatomically, the lower airway begins at the larynx (voice box). The cricoid cartilage is a firm cartilage ring that forms the inferior (lower) part of the larynx. The trachea is connected to the larynx and extends

Despite direct pressure, a large laceration continues to spurt large amounts of bright red blood. You should:
� A:elevate the extremity and apply a tight pressure dressing.
� B:place additional dressings on the wound until the bleeding stops.
� C:apply a

You selected C; This is correct!
Reason:
You must control any and all external bleeding as soon as possible. In the case of arterial bleeding (ie, bright red blood is spurting from the wound), the patient will bleed to death if immediate action is not tak

You respond to a shooting at a local bar. Law enforcement is present and the scene has been secured. Your patient is a young male, who is sitting against the wall screaming in pain. Bright red blood is spurting from a wound near his groin. You should:
� A

You selected B; This is correct!
Reason:
You must first address problems that pose the most immediate threat to life. The injury to the groin area most likely is an arterial bleed from the femoral artery; this bleeding must be controlled immediately or th

A 21-year-old man partially amputated his right arm when the chainsaw he was using to trim trees slipped. You can feel a weak radial pulse and his arm is cool to the touch. Dark red blood is flowing heavily from the wound. You should:
� A:control the blee

The correct answer is B;
Reason:
When caring for a partially amputated extremity, control bleeding with bulky compression (pressure) dressings and splint the extremity to prevent further injury. If direct pressure does not immediately control the bleeding

During your assessment of a trauma patient, you note massive facial injuries, weak radial pulses, and clammy skin. What should be your MOST immediate concern?
� A:Internal bleeding and severe shock
� B:Potential obstruction of the airway
� C:Providing rap

You selected B; This is correct!
Reason:
Any trauma patient with severe maxillofacial trauma is at an extremely high risk of airway compromise. The airway can be compromised by either mandibular fractures, in which the tongue may occlude the airway, or se

A soft-tissue injury that results in a flap of torn skin is called a/an:
� A:avulsion.
� B:incision.
� C:abrasion.
� D:laceration.

You selected A; This is correct!
Reason:
An avulsion is a soft-tissue injury in which a portion of the skin is torn away, leaving a flap of skin. A laceration is a jagged soft-tissue injury that can be caused by glass or other sharp objects. An abrasion i

An adult patient opens his eyes in response to a painful stimulus, moans when you ask him questions, and pulls his arm away when you palpate it. What is his Glasgow Coma Scale (GCS) score?
� A:7
� B:9
� C:8
� D:6

You selected C; This is correct!
Reason:
The Glasgow Coma Score (GCS) assesses three neurologic parameters: eye opening, verbal response, and motor response. Your patient's GCS score is 8. For eye opening, he receives 2 points for opening his eyes in resp

Which of the following is the preferred initial method for providing artificial ventilations to an apneic adult?
� A:Flow-restricted, oxygen-powered ventilation device
� B:Mouth-to-mask technique with supplemental oxygen
� C:One-person bag-valve-mask tech

You selected D; The correct answer is B;
Reason:
The preferred initial method for providing artificial ventilations is the mouth-to-mask technique with one-way valve and supplemental oxygen attached. Evidence has show that rescuers who ventilate patients

You arrive at the scene shortly after a 55-year-old man collapsed. Two bystanders are performing CPR. Your FIRST action should be to:
� A:stop CPR so you can assess breathing and pulse.
� B:attach the AED and analyze his cardiac rhythm.
� C:check the effe

You selected A; This is correct!
Reason:
Upon arriving at a scene where bystander CPR is in progress, you must first confirm that the patient is indeed apneic and pulseless and needs CPR. Bystanders who are not properly trained often perform CPR on patien

When ventilating an unresponsive apneic adult with a bag-mask device, you should ensure that:
� A:the pop-off relief valve is manually occluded.
� B:an airway adjunct has been inserted.
� C:ventilations occur at a rate of 20 breaths/min.
� D:you are posit

You selected B; This is correct!
Reason:
When ventilating an unresponsive apneic patient with a bag-mask device, you should ensure that an oral or nasal airway adjunct is inserted, which will keep the tongue off of the posterior pharynx. When ventilating

Which of the following airway sounds indicates a lower airway obstruction?
� A:Stridor
� B:Wheezing
� C:Gurgling
� D:Crowing

The correct answer is B;
Reason:
Wheezing is a whistling sound that results from narrowing and/or inflammation of the bronchioles in the lungs and is an indicator of a lower airway disease (ie, asthma, bronchiolitis). Crowing and stridor are both high-pit

Which of the following statements regarding artificial ventilation of an apneic patient who has dentures is correct?
� A:Tight-fitting dentures should be left in place because they facilitate the delivery of adequate tidal volume.
� B:Because of the risk

You selected A; This is correct!
Reason:
Providing artificial ventilation with a bag-mask or pocket face mask device is usually much easier when dentures can be left in place. Leaving the dentures in place provides "structure" to the face and will assist

A woman stabbed her boyfriend in the cheek with a dinner fork during an argument. Police have the woman in custody. The patient still has the fork impaled in his cheek. He is conscious and alert, breathing adequately, and has blood in his oropharynx. You

You selected C; This is correct!
Reason:
An impaled object in the cheek should be removed if it interferes with your ability to manage the patient's airway. In this case, however, the patient is breathing adequately and does not require aggressive airway

Ventilation is defined as the:
� A:movement of air into and out of the lungs.
� B:volume of air inhaled into the lungs in a single breath.
� C:elimination of carbon dioxide from the body.
� D:exchange of oxygen and carbon dioxide at the cell level.

The correct answer is A;
Reason:
Ventilation is defined as the movement of air into and out of the lungs. During negative-pressure ventilation (normal breathing), the diaphragm and intercostal muscles contract, which increases the vertical and horizontal

When caring for a critically injured patient, it is MOST appropriate to perform your secondary assessment:
� A:immediately after taking baseline vital signs.
� B:immediately following the primary assessment.
� C:after all life threats have been ruled out.

You selected D; This is correct!
Reason:
The secondary assessment is a detailed exam of the patient; it focuses on detecting and correcting injuries or conditions that were not grossly obvious during the primary assessment. Because the secondary assessmen

When assessing distal circulation in a patient with a swollen deformed femur, you should:
� A:touch his foot with a blunt object.
� B:palpate for a dorsalis pedis pulse.
� C:ask the patient to wiggle his toes.
� D:assess the pulse behind the knee.

The correct answer is B;
Reason:
Care for a musculoskeletal injury includes assessing distal circulatory, sensory, and motor functions before and after applying a splint. In the case of a femur injury, the dorsalis pedis (pedal) pulse, located on top of t

A 60-year-old woman presents with acute respiratory distress. She is conscious and alert, but restless. Her respiratory rate is 26 breaths/min with adequate chest expansion, her breath sounds are clear to auscultation bilaterally, and her oxygen saturatio

The correct answer is B;
Reason:
Although the patient is restless�a sign of hypoxemia�she is conscious and alert and able to maintain her own airway; therefore, an airway adjunct is not needed at this point. Furthermore, her respirations, although increas

Switching compressors during two-rescuer CPR:
� A:should take no more than 15 seconds to accomplish.
� B:is performed after every 10 to 20 cycles of adult CPR.
� C:is only necessary if the compressor becomes fatigued.
� D:should occur every 2 minutes thro

You selected D; This is correct!
Reason:
Rescuer fatigue may lead to inadequate chest compression rate and/or depth. Fatigue is common after 1 minute of CPR, although the rescuer may not recognize it for 5 minutes or longer. Therefore, compressors should

The myocardium receives its blood supply from the coronary arteries that branch directly from the:
� A:left atrium.
� B:right ventricle.
� C:aorta.
� D:vena cava.

You selected C; This is correct!
Reason:
The aorta, which is the largest artery in the human body, originates immediately from the left ventricle where it branches into the coronary arteries. This allows the myocardium to receive blood that has the highes

Which of the following describes the MOST appropriate method of performing chest compressions on an adult patient in cardiac arrest?
� A:Compress the chest at least 2", allow full recoil of the chest after each compression, minimize interruptions in chest

You selected A; This is correct!
Reason:
Effective chest compressions are essential for providing blood flow during CPR. To perform adequate chest compressions, the EMT should "push hard and push fast." Compress the chest of an adult at a rate of at least

A 60-year-old man is in cardiac arrest. You begin CPR while your partner applies the AED. What should you do if you receive a no shock message?
� A:Ensure that the AED electrodes are properly applied.
� B:Reanalyze his cardiac rhythm after 30 seconds of C

You selected D; This is correct!
Reason:
If the AED gives a no shock advised message, you should immediately resume CPR, starting with chest compressions. After 2 minutes of CPR, reanalyze the patient's cardiac rhythm and follow the AED voice prompts. You

Shallow respirations are an indication of:
� A:increased minute volume.
� B:increased carbon dioxide removal.
� C:increased oxygen intake.
� D:decreased tidal volume.

You selected D; This is correct!
Reason:
Shallow respirations are an indication of decreased tidal volume. Tidal volume is the amount of air (in milliliters [mL]) breathed into or out of the lungs in a single breath. Adequate tidal volume is needed to bri

Which of the following sets of vital signs is MOST suggestive of increased intracranial pressure in a patient who has experienced a traumatic brain injury?
� A:BP, 92/60 mm Hg; pulse, 120 beats/min; respirations, 24 breaths/min
� B:BP, 160/72 mm Hg; pulse

You selected D; This is correct!
Reason:
The body responds to a significant traumatic brain injury by shunting more oxygenated blood to the injured brain; it does this by increasing systemic blood pressure. In response to an increase in blood pressure, th

Which of the following questions would be MOST appropriate to ask when assessing a patient with chest pain?
� A:Does the pain radiate to your arm?
� B:What does the pain feel like?
� C:Is the pain worse when you take a deep breath?
� D:Would you describe

The correct answer is B;
Reason:
When questioning any patient about any type of pain, you should avoid asking leading questions that can simply be answered yes or no. To obtain the most reliable assessment, open-ended questions should be asked to allow th

Initial treatment for a patient with external blood loss depends upon:
� A:whether or not hypotension is present.
� B:his or her signs and symptoms.
� C:his or her past medical history.
� D:the amount of estimated blood loss.

The correct answer is B;
Reason:
It can be difficult to estimate the amount of external blood loss. This is because blood looks different on different surfaces, such as when it is absorbed by clothing or when it has been diluted with water. If possible, y

An elderly woman with COPD presents with a decreased level of consciousness, cyanosis to her face and neck, and labored respirations. Her pulse is rapid and weak and her oxygen saturation is 76%. You should:
� A:apply oxygen via nasal cannula and reassess

The correct answer is B;
Reason:
The patient in this scenario is experiencing an exacerbation (worsening) of her COPD. Her decreased level of consciousness; cyanosis; weak, rapid pulse; low oxygen saturation (SpO2); and labored breathing clearly indicate

You arrive at the scene of a 56-year-old man who collapsed. The patient's wife tells you that he suddenly grabbed his chest and then passed out. Your assessment reveals that he is apneic and pulseless. As your partner begins one-rescuer CPR, you should:

You selected A; This is correct!
Reason:
Immediate treatment for a patient in cardiac arrest involves performing CPR and applying the AED as soon as possible. After applying the AED pads to the patient's chest (around your partner's compressing hands), an

As you begin ventilating an unresponsive apneic man, you hear gurgling in his upper airway. Your MOST immediate action should be to:
� A:suction the patient's airway for no longer than 15 seconds.
� B:squeeze the bag-mask device with less force and reasse

The correct answer is D;
Reason:
Gurgling in the airway indicates the presence of vomitus or other secretions. If this is noted, you should immediately turn the patient onto his side to allow the secretions to drain. After placing the patient on his side,

A 22-year-old man was stabbed in the chest with a large knife. The patient is pulseless and apneic, and the knife is impaled in the center of his chest. Treatment should include:
� A:stabilizing the knife, starting CPR, and providing rapid transport.
� B:

The correct answer is C;
Reason:
There are two indications for removing an impaled object: when the object is causing airway compromise and when the object interferes with your ability to perform CPR. A knife impaled in the center of the chest, which is w

A 19-year-old female has a closed, swollen deformity to her left forearm. You are unable to palpate a radial pulse and the skin distal to the injury is cold and pale. Several attempts to contact medical control have failed and you are approximately 45 mil

The correct answer is D;
Reason:
Cold, pale skin and an absent distal pulse indicates that blood flow distal to the injury is compromised. You should notify medical control, who will likely direct you to attempt to restore distal circulation. However, if

Which of the following is the MOST correct technique for ventilating an apneic adult who has a pulse?
� A:Deliver each breath over 1 second at a rate of 10 to 12 breaths/min.
� B:Hyperventilate at a rate between 20 and 24 breaths/min.
� C:Ventilate at a r

The correct answer is A;
Reason:
When ventilating an apneic adult who has a pulse, deliver each breath over a period of 1 second, at a rate of 10 to 12 breaths/min (one breath every 5 to 6 seconds), while observing for visible chest rise. A ventilation ra

During your assessment of a 70-year-old man with crushing chest pain, you note that his blood pressure is 80/50 mm Hg. Your MOST important action should be to:
� A:assess his oxygen saturation.
� B:give high-flow oxygen.
� C:transport without delay.
� D:k

You selected C; This is correct!
Reason:
Patients with chest pain, pressure, or discomfort with a systolic BP less than 100 mm Hg should be transported to the hospital without delay. Hypotension in a patient with chest pain indicates cardiogenic shock due

Hypoxia is defined as:
� A:inadequate oxygen to the body's cells and tissues.
� B:an excess amount of carbon dioxide in arterial blood.
� C:decreased oxygen content in arterial blood.
� D:an absence of oxygen to the vital body organs.

You selected A; This is correct!
Reason:
Hypoxia is a dangerous condition in which the body's cells and tissues do not have enough oxygen. Hypoxemia is a decreased amount of oxygen in arterial blood. Untreated hypoxemia will lead to hypoxia. An absence of

After an adult cardiac arrest patient has been intubated by a paramedic, you are providing ventilations as your partner performs chest compressions. When ventilating the patient, you should:
� A:deliver each breath over 2 seconds at a rate of 12 to 15 bre

The correct answer is D;
Reason:
When ventilating an adult cardiac arrest patient with an advanced airway in place (ie, ET tube, multilumen airway, supraglottic airway), you should deliver each breath over a period of 1 second�just enough to produce visib

Which of the following injury mechanisms is associated with hangings?
� A:Subluxation
� B:Axial loading
� C:Hyperextension
� D:Distraction

The correct answer is D;
Reason:
Injury to the cervical spine following a hanging occurs via distraction, or stretching, of the vertebrae and spinal cord. A subluxation is a partial or incomplete dislocation; it is an injury, not an injury mechanism. Inju

Prior to applying a nonrebreathing mask on a patient with difficulty breathing, you should:
� A:set the flow rate to no more than 10 liters per minute.
� B:perform a complete exam to assess the degree of hypoxia.
� C:insert a nasopharyngeal airway to main

You selected D; This is correct!
Reason:
After attaching the nonrebreathing mask to the oxygen source, the flowmeter should be set to between 12 and 15 L/min. The reservoir bag is then prefilled with oxygen, which will allow the delivery of high-flow oxyg

During your assessment of an unresponsive adult female, you determine that she is apneic. You should:
� A:place an oropharyngeal airway.
� B:deliver two rescue breaths.
� C:begin chest compressions.
� D:assess for a carotid pulse.

You selected D; This is correct!
Reason:
As soon as you determine that an adult patient is apneic or only has agonal gasps, you should assess for a carotid pulse for at least 5 seconds but no more than 10 seconds. If the patient has a pulse, provide rescu

A 40-year-old man has burns to the entire head, anterior chest, and both anterior upper extremities. Using the adult Rule of Nines, what percentage of his total body surface area has been burned?
� A:18%
� B:45%
� C:36%
� D:27%

You selected D; This is correct!
Reason:
Using the adult Rule of Nines, the head accounts for 9% of the total body surface area (TBSA), the anterior chest for 9% (the entire anterior trunk [chest and abdomen] accounts for 18%), and the anterior upper extr

Which of the following statements regarding ventricular fibrillation (V-Fib) is correct?
� A:Any patient in V-Fib must receive CPR for 2 minutes prior to defibrillation.
� B:Patients in V-Fib should be defibrillated after every 60 seconds of CPR.
� C:In V

The correct answer is C;
Reason:
Ventricular fibrillation (V-Fib) is a disorganized, ineffective quivering of the heart muscle. No blood is pumped through the body and the patient is pulseless. Loss of consciousness occurs within seconds following the ons

A patient with a closed head injury opens his eyes in response to pain, is mumbling words that you cannot understand, and pushes your hand away when you apply a painful stimulus. His Glasgow Coma Scale (GCS) score is:
� A:6
� B:8
� C:7
� D:9

The correct answer is D;
Reason:
The Glasgow Coma Scale (GCS) is a valuable tool used when assessing patients with a neurological injury. It assesses three parameters: eye opening, verbal response, and motor response. A minimum score on the GCS is 3 and a

After restoring a pulse in a cardiac arrest patient, you begin immediate transport. While en route to the hospital, the patient goes back into cardiac arrest. You should:
� A:begin CPR and proceed to the hospital.
� B:tell your partner to stop the ambulan

You selected B; This is correct!
Reason:
If you restore a pulse in a cardiac arrest patient, the AED pads should remain attached to the patient's chest during transport in case cardiac arrest recurs en route to the hospital. You should, however, turn the

To obtain the MOST reliable assessment of a patient's tidal volume, you should:
� A:look at the rise of the chest.
� B:assess for retractions.
� C:count the respiratory rate.
� D:listen for airway noises.

You selected A; This is correct!
Reason:
Tidal volume is the amount of air, in milliliters, breathed into or out of the lungs in a single breath. The most effective (and practical) way to assess tidal volume is to evaluate the rise of the patient's chest.

The MOST effective way to determine if you are providing adequate volume during artificial ventilation is:
� A:assessing the chest for adequate rise.
� B:checking the skin for improvement of cyanosis.
� C:assessing the pulse for an improving heart rate.

You selected A; This is correct!
Reason:
The goal of providing artificial ventilation is to provide adequate tidal volume to the patient so that enough oxygen is delivered to the lungs, and ultimately, the cells of the body. The most effective way to dete

What is the function of pulmonary surfactant?
� A:It facilitates the production of mucous, which is expelled during coughing.
� B:It carries fresh oxygen from the lungs to the left side of the heart.
� C:It lubricates the alveolar walls and allows them to

You selected C; This is correct!
Reason:
Surfactant is a lubricant that lines the alveolar walls. It allows them to expand and recoil freely, thereby allowing for an easy exchange of oxygen and carbon dioxide. Diseases such as emphysema cause destruction

A young male has an open abdominal wound through which a small loop of bowel is protruding. There is minimal bleeding. The BEST way to treat his injury is to:
� A:cover the wound with a dry sterile trauma dressing and tightly secure it in place by circumf

You selected B; This is correct!
Reason:
An abdominal evisceration occurs when a loop of bowel, an organ, or fat protrudes through an open abdominal injury. Never try to replace an organ that is protruding from an open abdominal wound, whether it is a sma

A patient who is breathing with reduced tidal volume would MOST likely have:
� A:a prolonged inhalation phase.
� B:warm, moist skin.
� C:a respiratory rate of 14 breaths/min.
� D:shallow respirations.

You selected D; This is correct!
Reason:
Tidal volume, a measure of the depth of breathing, is the amount of air (in milliliters [mL]) that is moved into or out of the lungs during a single breath; in the average adult male, this is about 500 mL. Tidal vo

Which of the following would clearly be detrimental to a patient in cardiac arrest?
� A:Ventilating just until the chest rises
� B:Using a pocket face mask without high-flow oxygen
� C:Interrupting CPR for more than 10 seconds
� D:Performing CPR before de

You selected C; This is correct!
Reason:
Major emphasis is placed on minimizing interruptions in CPR. Even brief interruptions cause a significant decrease in blood flow to the heart and brain. If you must interrupt CPR, do not exceed 10 seconds. It is pr

Which of the following actions is MOST important when immobilizing a patient with a suspected spinal injury?
� A:Select and apply the appropriate size of extrication collar.
� B:A vest-style immobilization device should routinely be used.
� C:Check range

The correct answer is A;
Reason:
Although an extrication (cervical) collar is not the sole means of immobilizing the patient's spine, it must be of the appropriate size in order to minimize flexion/extension of the patient's neck. When immobilizing any pa

A middle-aged woman took three of her prescribed nitroglycerin tablets after she began experiencing chest pain. She complains of a bad headache and is still experiencing chest pain. You should assume that:
� A:her chest pain is not cardiac-related.
� B:he

You selected C; This is correct!
Reason:
A headache and/or a bitter taste under the tongue are common side effects of nitroglycerin (NTG) that many patients experience. If the patient does not experience these side effects, the NTG may have lost its poten

The position of comfort for a patient with nontraumatic chest pain MOST commonly is:
� A:supine with the legs elevated slightly.
� B:on the side with the head elevated.
� C:semisitting.
� D:lateral recumbent.

You selected C; This is correct!
Reason:
As with most patients, the position of comfort for cardiac patients typically is the semisitting (semi-Fowler) position. You should allow the patient to remain in the position of comfort both during the assessment

How should you treat an unresponsive, uninjured patient with respirations of 16 breaths/min and good chest expansion?
� A:Airway adjunct and oxygen via nonrebreathing mask
� B:Suctioning as needed and artificial ventilations
� C:Jaw-thrust maneuver and fr

You selected A; This is correct!
Reason:
After opening the airway of an unresponsive patient, an airway adjunct (oral or nasal airway) should be inserted to keep the tongue from occluding the posterior pharynx. Oral and nasal airways are used in conjuncti

Which of the following processes occurs during inhalation?
� A:The diaphragm contracts and the intercostal muscles relax.
� B:The intercostal muscles relax and the diaphragm descends.
� C:The intercostal muscles and diaphragm both contract.
� D:The diaphr

The correct answer is C;
Reason:
During the active process of inhalation, the diaphragm contracts, causing it to descend. This increases the vertical dimensions of the chest. At the same time, the intercostal muscles (muscles between the ribs) contract, i

When suctioning copious secretions from a semiconscious adult's airway, you should:
� A:avoid touching the back of the airway with the suction catheter.
� B:suction for up to 20 seconds while withdrawing the catheter.
� C:use a flexible catheter because i

You selected A; This is correct!
Reason:
When suctioning a patient's airway, especially if he or she is semiconscious, you should avoid touching the back of the airway with the suction catheter. Inserting the catheter too far may stimulate the gag reflex,

A patient experienced blunt chest trauma and has asymmetrical chest wall movement. This MOST likely indicates:
� A:accumulation of blood in both of the lungs.
� B:shallow breathing secondary to severe pain.
� C:decreased air movement into one lung.
� D:se

You selected C; This is correct!
Reason:
Asymmetrical chest wall movement, when one side of the chest moves less than the other, indicates decreased air movement into one lung (eg, pneumothorax, hemothorax). Bleeding into both lungs and shallow breathing

Signs of inadequate breathing in an unresponsive patient include:
� A:an irregular pulse.
� B:warm, moist skin.
� C:symmetrical chest rise.
� D:cyanotic oral mucosa.

You selected D; This is correct!
Reason:
Signs of inadequate breathing in both responsive and unresponsive patients include a respiratory rate that is too slow (less than 12 breaths/min) or too fast (greater than 20 breaths/min); shallow (reduced tidal vo

A middle-aged female with a history of hypertension and high cholesterol complains of chest discomfort. She asks you to take her to the hospital where her personal physician practices, which is 15 miles away. Her blood pressure is 130/70 mm Hg, pulse is 8

The correct answer is B;
Reason:
You should NEVER allow a patient with a possible cardiac problem to walk to the ambulance. This causes exertion, which increases cardiac oxygen consumption and demand and could worsen his or her condition. Give the patient

A patient overdosed on several drugs and is unresponsive with shallow breathing and facial cyanosis. As you continue your assessment, the patient suddenly vomits. You should:
� A:insert an oropharyngeal airway.
� B:begin assisting his ventilations.
� C:su

The correct answer is D;
Reason:
The patient's airway must be clear of foreign bodies or secretions before it can be assessed or managed. If the patient begins to vomit, he must first be rolled onto his side to allow for drainage of the vomitus. Use sucti

Which of the following ventilation techniques will enable you to provide the greatest tidal volume AND allow you to effectively assess lung compliance?
� A:One-rescuer bag-mask ventilation
� B:Flow-restricted, oxygen-powered ventilation
� C:One-rescuer mo

You selected C; This is correct!
Reason:
Because the EMT uses both of his or her hands to obtain a mask seal, the one-rescuer mouth-to-mask ventilation technique will provide the greatest tidal volume compared to the other methods listed. Furthermore, lun

A 21-year-old male was bitten on the left forearm by a dog. He is conscious and alert and denies any other injuries. An animal control officer is at the scene and has contained the dog. Your assessment of the patient's arm reveals a large avulsion with a

The correct answer is C;
Reason:
An avulsion is an injury that separates various layers of soft tissue, usually between the subcutaneous layer and fascia, so that they become completely detached or hang as a flap. The patient's injury is isolated and not

Which of the following devices is contraindicated in patients with blunt chest trauma?
� A:Oral airway
� B:Nasal airway
� C:Oxygen-powered ventilator
� D:Bag-mask device

You selected C; This is correct!
Reason:
The flow-restricted, oxygen-powered ventilation device (FROPVD), also referred to as an oxygen-powered ventilator or manually-triggered ventilator, should not be used in patients with chest trauma; it delivers oxyg

Which of the following statements regarding one-rescuer CPR is correct?
� A:Ventilations should be delivered over a period of 2 to 3 seconds.
� B:The chest should be allowed to fully recoil after each compression.
� C:You should assess the patient for a p

You selected B; This is correct!
Reason:
When performing CPR on any patient, you should allow the chest to fully recoil after each compression. Incomplete chest recoil causes increased intrathoracic pressure, which may impair blood return to the heart. As

Which of the following mechanisms of injury would necessitate performing a rapid head-to-toe assessment?
� A:A 5 foot, 9 inch tall adult who fell 12 feet from a roof and landed on his side
� B:An impaled object in the patient's lower extremity with minima

You selected D; This is correct!
Reason:
Significant mechanisms of injury include, among others, falls of greater than 15 feet (or three times the patient's height), penetrating injuries to the trunk and head, high-speed motor vehicle crashes, rollover mo

Which of the following patients would be the LEAST likely to present with classic signs and symptoms of acute myocardial infarction?
� A:55-year-old female with COPD and frequent infections.
� B:59 year-old male with alcoholism and angina pectoris.
� C:72

The correct answer is C;
Reason:
Chest pain, pressure, or discomfort (usually lasting greater than 15 minutes) is present in the majority of patients experiencing acute myocardial infarction (AMI). Other common signs and symptoms include shortness of brea

A patient who is experiencing an acute myocardial infarction:
� A:has chest pain or discomfort that does not change with each breath.
� B:often complains of a different type of pain than a patient with angina.
� C:most often describes his or her chest pai

You selected A; This is correct!
Reason:
The type of chest pain or discomfort associated with acute myocardial infarction (AMI) is the same that is experienced by patients with angina pectoris (eg, dull, crushing, pressure, heaviness); thus, you cannot di

Following a head injury, a young female is semiconscious and is bleeding from the nose and left ear. You should:
� A:place a pressure dressing over her ear to prevent blood loss.
� B:insert a nasal airway to keep her tongue from blocking the airway.
� C:c

You selected D; The correct answer is C;
Reason:
Blood draining from the ears or nose following a head injury may contain cerebrospinal fluid (CSF) and indicates a skull fracture. In these cases, do NOT attempt to stop the flow of blood. Applying excessiv

Nitroglycerin is contraindicated in all of the following situations, EXCEPT:
� A:systolic BP of 80 mm Hg.
� B:recent use of Cialis.
� C:history of cardiac bypass surgery.
� D:the presence of a head injury.

You selected C; This is correct!
Reason:
Nitroglycerin (NTG) is a vasodilator drug used to relieve chest pain in patients with cardiac compromise by dilating the coronary arteries and improving blood flow to the heart. Because of its vasodilator effects,

Which of the following describes pulseless electrical activity (PEA)?
� A:A rapid cardiac rhythm that does not produce a pulse, but responds to defibrillation
� B:Any organized cardiac rhythm, slow or fast, that does not produce a palpable pulse
� C:The p

The correct answer is B;
Reason:
Pulseless electrical activity (PEA) is a condition in which the heart produces organized electrical activity (slow or fast), despite the absence of a palpable pulse. A disorganized, chaotic quivering of the heart muscle th

Which of the following questions would be the MOST effective in determining if a patient's chest pain radiates away from his or her chest?
� A:Do you also have pain in your arm, jaw, or back?
� B:Is there any other part of your body where you have pain?

You selected C; This is correct!
Reason:
When assessing a patient with any type of pain, you should avoid asking leading questions; instead, ask open-ended questions whenever possible. For example, instead of asking the patient if his or her pain is dull,

Occasional, irregular breaths that may be observed in a cardiac arrest patient are called:
� A:Biot respirations.
� B:Cheyne-Stokes respirations.
� C:ataxic respirations.
� D:agonal gasps.

You selected D; This is correct!
Reason:
Occasional, irregular breaths, called agonal gasps, may be observed in some patients shortly after their heart stops beating. They occur when the respiratory center in the brain sends stray signals to the respirato

While transporting an elderly woman who was complaining of nausea, vomiting, and weakness, she suddenly becomes unresponsive. You should:
� A:feel for a carotid pulse for at least 5 seconds.
� B:analyze her heart rhythm with the AED.
� C:open her airway a

The correct answer is D;
Reason:
If a patient is found unresponsive or becomes unresponsive in your presence, your first action should be to assess for breathing; this should be done by quickly (no more than 10 seconds) looking at the chest for obvious mo

You should suspect that your patient has pulmonary edema if he or she:
� A:has swollen feet and ankles.
� B:has a dry, nonproductive cough.
� C:is hypertensive and tachycardic.
� D:cannot breathe while lying down.

You selected D; This is correct!
Reason:
Pulmonary edema is often caused by failure of the left side of the heart. When the patient is lying down, he or she experiences worsened difficulty breathing (orthopnea) because more blood backs up in the lungs. Pa

All of the following are components of the Cincinnati stroke scale, EXCEPT:
� A:pupil size.
� B:arm movement.
� C:facial droop.
� D:speech pattern.

The correct answer is A;
Reason:
The Cincinnati Stroke Scale, which tests speech, facial droop, and arm drift, is a reliable tool that should be used during your assessment of a patient suspected of having had a stroke. To test speech, ask the patient to

Despite direct pressure, a large laceration to the medial aspect of the arm continues to bleed profusely. You should:
� A:pack the inside of the laceration with sterile gauze.
� B:locate and apply pressure to the brachial artery.
� C:continue direct press

You selected D; This is correct!
Reason:
In most cases, external bleeding can be controlled with direct pressure. However, if a wound continues to bleed profusely despite direct pressure, a proximal tourniquet should be applied without delay. If the exter

The wall that separates the left and right sides of the heart is called the:
� A:pericardium.
� B:carina.
� C:mediastinum.
� D:septum.

The correct answer is D;
Reason:
The septum is the wall that separates the left and right sides of the heart. There is a septum for both the atria and the ventricles. The carina is the bifurcation point of the trachea, and the mediastinum is the space bet

At the peak of the inspiratory phase, the alveoli in the lungs contain:
� A:high quantities of carbon dioxide.
� B:equal levels of oxygen and carbon dioxide.
� C:minimal levels of oxygen and carbon dioxide.
� D:more oxygen than carbon dioxide.

You selected D; This is correct!
Reason:
At the peak of the inspiratory (inhalation) phase, the alveoli are filled with fresh oxygen that the patient just breathed in. During the expiratory (exhalation) phase, the oxygen moves from the alveoli to the left

During a soccer game, a 20-year-old man collided shoulder-to-shoulder with another player. He has pain and a noticeable anterior bulge to the left shoulder. Which of the following is the MOST effective method of immobilizing this injury?
� A:An air-inflat

You selected C; This is correct!
Reason:
Injuries to the shoulder are most effectively immobilized with the use of a sling and swathe. The sling will provide support and relieve pain to the shoulder, and the swathe will secure the arm to the body. The pur

If a vehicle strikes a tree at 60 MPH, the unrestrained driver would likely experience the MOST severe injuries during the:
� A:first collision.
� B:fourth collision.
� C:third collision.
� D:second collision.

You selected C; This is correct!
Reason:
Motor-vehicle crashes typically consist of three separate collisions. Understanding the events that occur during each collision will help you remain alert for certain types of injury patterns. During the first coll

The MOST appropriate treatment for a semiconscious patient with slow, shallow respirations includes:
� A:an oropharyngeal airway and assisted ventilation with a bag-mask device.
� B:a nasopharyngeal airway and assisted ventilation with a bag-mask device.

You selected B; This is correct!
Reason:
Semiconscious patients are not fully able to protect their own airway and require an airway adjunct. The nasopharyngeal airway is indicated for semiconscious patients because they often have an intact gag reflex; t

The energy setting for a biphasic AED:
� A:increases by 50 joules with each shock.
� B:must be manually set by the EMT.
� C:is manufacturer specific.
� D:is typically 360 joules.

The correct answer is C;
Reason:
A biphasic AED sends energy in two directions simultaneously. This is advantageous because it produces a more efficient defibrillation than monophasic defibrillation, which sends energy in only one direction, and uses a lo

A 58-year-old man is found unresponsive by his wife. According to his wife, he was complaining of a "dull ache" in his chest the day before, but refused to allow her to call 9-1-1. His blood pressure is 70/50 mm Hg, his pulse is 120 beats/min and weak, an

You selected B; This is correct!
Reason:
The patient in this scenario likely experienced an acute myocardial infarction the day before. However, because he refused medical care, the infarction has likely damaged a significant portion of his heart, resulti

In which of the following circumstances would external bleeding be the LEAST difficult to control?
� A:Lacerated brachial artery; BP of 140/90 mm Hg
� B:Lacerated femoral vein; BP of 70/40 mm Hg
� C:Lacerated carotid artery; BP of 90/50 mm Hg
� D:Lacerate

The correct answer is B;
Reason:
It is generally less difficult to control external bleeding from a lacerated vein rather than an artery. Unlike arteries, veins are under low pressure. Furthermore, the presence of a low blood pressure (hypotension), which

The automated external defibrillator (AED) should NOT be used in patients who:
� A:have a nitroglycerin patch applied to the skin.
� B:are between 1 and 8 years of age.
� C:experienced a witnessed cardiac arrest.
� D:are apneic and have a weak carotid pul

The correct answer is D;
Reason:
The AED is only applied to patients in cardiac arrest (eg, pulseless and apneic), whether the arrest was witnessed or unwitnessed. According to the 2010 guidelines for CPR and Emergency Cardiac Care (ECC), AEDs can safely

Appropriate care for an amputated body part includes all of the following, EXCEPT:
� A:keeping the part cool, but not allowing it to freeze.
� B:placing it directly on ice to prevent tissue damage.
� C:wrapping it in a sterile dressing and placing it in a

You selected B; This is correct!
Reason:
Appropriate care for an amputated body part includes wrapping the part in a sterile dressing and placing it in a plastic bag. Follow your local protocols regarding how to preserve amputated parts. In some areas, dr

A 42-year-old man was ejected from his car after it struck a bridge pillar at a high rate of speed. You find him in a prone position approximately 50 feet from his car. He is not moving and does not appear to be breathing. You should:
� A:manually stabili

You selected A; This is correct!
Reason:
When a trauma patient is found in a prone (face-down) position, especially if he or she is unresponsive, your first action should be to manually stabilize his or her head; this action is based on the assumption tha

Which of the following is MOST indicative of compensated shock in an adult?
� A:Weak carotid pulse, cool skin, increased respiratory rate
� B:Unresponsive, pallor, absent radial pulses, tachypnea
� C:Restless, diaphoresis, tachypnea, BP of 104/64 mm Hg

You selected C; This is correct!
Reason:
In compensated shock, the nervous system is mounting a physiologic response to an underlying illness or injury in order to maintain perfusion to vital organs such as the brain, heart, and kidneys. The patient with

Following administration of nitroglycerin to a man with crushing chest pressure, he experiences a significant increase in his heart rate. This is MOST likely the result of:
� A:a cardiac dysrhythmia.
� B:a drop in blood pressure.
� C:coronary vasoconstric

The correct answer is B;
Reason:
Nitroglycerin (NTG) is a vasodilator drug. It dilates not only the coronary arteries, but also other arteries in the body. In some patients, NTG may cause a drop in blood pressure, especially if they are taking other medic

The process of loading oxygen molecules onto hemoglobin molecules in the bloodstream is called:
� A:respiration.
� B:oxygenation.
� C:ventilation.
� D:diffusion.

You selected B; This is correct!
Reason:
Oxygenation is the process of loading oxygen molecules onto hemoglobin molecules in the blood. Adequate oxygenation is required for internal (cellular) respiration to take place. Diffusion is the process in which g

Which of the following patients would MOST likely present with vague or unusual symptoms of an acute myocardial infarction?
� A:75-year-old male with hypertension
� B:55-year-old obese female
� C:66-year-old male with angina
� D:72-year-old female with di

You selected D; This is correct!
Reason:
Not all patients experiencing acute myocardial infarction (AMI) present with the classic signs and symptoms one would expect. Middle-aged men often minimize their symptoms and attribute their chest pain or discomfo

Which of the following is MOST indicative of a primary cardiac problem?
� A:Tachypnea
� B:Sudden fainting
� C:Irregular pulse
� D:Tachycardia

You selected C; This is correct!
Reason:
An irregular pulse signifies an abnormality within the electrical conduction system of the heart. Tachycardia, sudden fainting (syncope), and tachypnea (rapid breathing) can indicate many things other than cardiac

A 56-year-old man was the unrestrained driver of a small passenger car that rolled over twice after he rounded a corner too fast. He is unresponsive; has rapid, shallow respirations; and has a rapid, weak pulse. His left arm is completely amputated just b

You selected D; This is correct!
Reason:
Life takes priority over limb. The patient is in shock, which may be complicated by a head injury; therefore, he requires rapid transport to a trauma center. Although efforts should be made to recover an amputated

When assessing a patient who complains of chest pain, which of the following questions would you ask to assess the "R" in OPQRST?
� A:Is there anything that makes the pain worse?
� B:Is the pain in one place or does it move around?
� C:Did the pain begin

You selected B; This is correct!
Reason:
The "R" in OPQRST stands for radiation or referred pain. An appropriate way to determine whether the pain radiates or not is to ask the patient if the pain remains in one place or if it moves around. When determini

You are assessing a young male who was stabbed in the right lower chest. He is semiconscious and has labored breathing, collapsed jugular veins, and absent breath sounds on the right side of his chest. This patient MOST likely has a:
� A:liver laceration.

You selected B; This is correct!
Reason:
You should suspect a hemothorax if a patient with chest trauma presents with shock, especially if the injury was caused by penetrating trauma. Hemothorax occurs when blood collects in the pleural space and compress

A reduction in tidal volume would MOST likely result from:
� A:increased minute volume.
� B:unequal chest expansion.
� C:flaring of the nostrils.
� D:accessory muscle use.

You selected B; This is correct!
Reason:
Unequal (asymmetrical) or minimal expansion of the chest results in a decrease in the amount of air inhaled per breath (tidal volume). Accessory muscle use and nasal flaring are signs of increased work of breathing

Medications such as albuterol (Ventolin) relieve respiratory distress by:
� A:contracting the smaller airways in the lungs.
� B:dilating the large mainstem bronchi of the airway.
� C:constricting the bronchioles in the lungs.
� D:relaxing the smooth muscl

You selected D; This is correct!
Reason:
Medications such as albuterol (Ventolin) and metaproterenol (Alupent) are in a class of drugs called bronchodilators. They relax the smooth muscle found within the bronchioles in the lungs, which causes them to dil

During your assessment of a patient with a gunshot wound to the chest, you note that his skin is pale. This finding is the result of:
� A:decreased blood flow to the skin.
� B:a significantly elevated heart rate.
� C:a critically low blood pressure.
� D:p

You selected A; This is correct!
Reason:
When the body attempts to compensate for shock, peripheral vasoconstriction shunts blood away from the skin to the more vital organs in the body such as the brain, heart, lungs, and kidneys. When there is minimal o

When ventilating an apneic patient, you note decreased ventilatory compliance. This means that:
� A:the lungs are difficult to ventilate.
� B:the upper airway is blocked.
� C:you meet no resistance when ventilating.
� D:fluid is occupying the alveoli.

You selected A; This is correct!
Reason:
As it applies to artificial ventilation, compliance is the ability of the lungs to expand during ventilation. Increased ventilatory compliance means that no resistance is met when you ventilate the patient; you can

A 22-year-old female woman was shot by her husband. Law enforcement is at the scene and has the husband in custody. The patient is conscious, but extremely restless, and is pale and diaphoretic. As your partner administers high-flow oxygen, you should:

You selected D; This is correct!
Reason:
The primary assessment of any patient includes ensuring a patent airway, assessing breathing adequacy, administering high-flow oxygen or assisting ventilations, assessing circulation, and controlling all active ble

A 30-year-old man sustained partial-thickness burns to the anterior chest and both anterior arms. Based on the Rule of Nines, what percentage of his body surface area has been burned?
� A:36%
� B:9%
� C:18%
� D:27%

The correct answer is C;
Reason:
According to the adult Rule of Nines, the anterior trunk (chest and abdomen) accounts for 18% of the total body surface area (TBSA) and each entire arm accounts for 9%. Therefore, the anterior chest, which is one half of t

Snoring respirations in an unresponsive patient are usually the result of:
� A:swelling of the upper airway structures.
� B:foreign body airway obstruction.
� C:collapse of the trachea during breathing.
� D:upper airway obstruction by the tongue.

You selected D; This is correct!
Reason:
In an unresponsive patient, the muscles of the tongue, which attach to the mandible, relax and fall back over the posterior pharynx. This makes obstruction by the tongue the most common cause of airway obstruction